PLE Physio Part 1

¡Supera tus tareas y exámenes ahora con Quizwiz!

Brown-Sequard Syndrome will lead to contralateral ___ and ipsilateral ____

(+) contralateral loss of pain and temperature sensation beginning 1-2 segments below the lesion (+) ipsilateral weakness and spasticity in certain muscles groups

Which of the following is a property of C fibers? (A) Have the slowest conduction velocity of any nerve fiber type (B) Have the largest diameter of any nerve fiber type (C) Are afferent nerves from muscle spindles (D) Are afferent nerves from Golgi tendon organs (E) Are preganglionic autonomic fibers

(A) Have the slowest conduction velocity of any nerve fiber type - C fibers (B) Have the largest diameter of any nerve fiber type - A fibers (C) Are afferent nerves from muscle spindles - 1a fibers (D) Are afferent nerves from Golgi tendon organs - 1b fibers (E) Are preganglionic autonomic fibers - B fibers

Adenosine triphosphate (ATP) is used indirectly for which of the following processes? (A) Accumulation of Ca2+ by the sarcoplasmic reticulum (SR) (B) Transport of Na+ from intracellular to extracellular fluid (C) Transport of K+ from extracellular to intracellular fluid (D) Transport of H+ from parietal cells into the lumen of the stomach (E) Absorption of glucose by intestinal epithelial cells

(E) Absorption of glucose by intestinal epithelial cells Because all secondary active transports (e.g. SGLT-1), relies on the Na+-gradient created by the Na+-K+-ATPase pump.

Alveolar Ventilation formula

(tidal volume - dead space) x respiratory rate Alveolar Ventilation is also called CORRECTED MINUTE VENTILATION. Its formula is basically minute ventilation minus physiologic dead space.

in muscle contraction, partial hydrolysis of ATP will lead to ____ and complete hydrolysis of ATP will lead to ____

* partial hydrolysis of ATP -> "recocking" of the myosin heads -> ADP produced Myosin then binds to Second Binding Site in actin. * complete hydrolysis of ADP -> "power / force-generating stroke" This shortens the sarcomere by 10Nm

Assuming complete dissociation of all solutes, which of the following solutions would be hyperosmotic to 1 mM NaCl? (A) 1 mM glucose (B) 1.5 mM glucose (C) 1 mM CaCl2 (D) 1 mM sucrose (E) 1 mM KCl

1 mM CaCl2 1mM x 3

muscle contraction steps

1) Motor neuron releases acetylcholine into the neuromuscular junction and causes the depolarization of the sarcolemma. 2) Depolarization spreads down the sarcolemma to the T-tubules, triggering the release of Ca2+ ions. 3) Ca2+ binds to troponin, causing a shift in tropomyosin and exposure of the myosin-binding site on the actin filament. 4) Shortening of the sarcomere occurs as the myosin head binds to the exposed sites of actin, forming a cross-bridge and pulling the actin filament along the thick filament, resulting in contraction. 5) Muscle relaxes when acetylcholine is degraded by acetylcholine esterase and the allowing Ca2+ is brought back into the SR. ATP binds to myosin head, allowing it to relax from actin.

WHAT ARE 3 SPECIAL CHARACTERISTICS OF OLFACTION/OLFACTORY NERVE

1. Only Neurons capable of reproduction 2. uses Type C nerve fibers (the slowest, least precise and accurate nerve fiber) - CN5 3. it is the only sensory modality that does NOT send fibers to the thalamus

action potential 3 characteristics

1. Stereotypical size and shape (meaning if I graph it, I will get the same thing again and again) 2. propagation (kumakalat - pag nag AP ang isang cell, magkakaroon ng AP yung next cell - di mababawasan yung lakas, NONDECREMENTAL) 3. all-or-none ("on" or "off" state. It will be in the "on" state once threshold is reached.)

A 35-year old man has a vital capacity (VC) of 5 L, a tidal volume (TV) of 0.5 L, an inspiratory capacity of 3.5 L, and a functional residual capacity (FRC) of 2.5 L. What is his expiratory reserve volume (ERV)?

1.5 L

ratio of arterial compliance to venous compliance

1:20 Capacitance of Veins > Arteries Capacitance of Arteries decreases with age

HEART BLOCK 1st - 3rd degree

1st degree AV Block: all atrial impulses reach the ventricles, but PR interval is long 2nd degree AV Block: no all impulses conducted to ventricles, ventricular rate < atrial rate. P Wave NOT always followed by QRS - dropped QRS complex 3rd degree (Complete) AV Block: Atrioventricular dissociation may cause fainting, syncope, worsening exercise intolerance from cerebral ischemia - Analogy: Parang stages sa break-up yan. 1st degree AV block - para kayong bf-gf na nagkakalaboan na; nagiging malayo na feelings nyo sa isa't isa (prolonged PR interval). 2nd degree: you drop dates and meetings na for one another (dropped QRS complex). 3rd degree: hiwalayan na talaga - kanya-kanyang buhay na (AV dissociation).

Lung volumes and Capacities: male vs female

20-25% lower in females

Start of Surfactant production? Maturation of Surfactant? Test for Surfactant?

24TH WEEK AOG 35TH WEEK AOG AMNIOTIC L:S RATIO

In a hospital error, a 60-year-old woman is infused with large volumes of a solution that causes lysis of her red blood cells (RBCs). The solution was most likely: (A) 150 mM NaCl (B) 300 mM mannitol (C) 350 mM mannitol (D) 300 mM urea (E) 150 mM CaCl2

300 mM urea A and B are isotonic compared to the RBC ICF (yung A, kaya naging isotonic, remember na magseseparate yung Na and Cl, kaya and actual effective osmotic pressure nyan ay 150 x 2 = 300. C and E are hypertonic (C is hypertonic dahil 150 x 3 naman). Dis hypotonic - not because of the concentration na 300, but because ang RC niya is less than 1 (ineffective osmole siya, so ang effective osmotic pressure niya is actually less than 300). Because D is hypotonic, water will move from ECF to ICF causing cellular swelling, and eventually, cellular rupture.

In a capillary, Pc is 30mmHg, Pi is -2mmHg, πc is 25mmHg, and πi is 2mmHg. If Kf is 0.5 mL/min/mm Hg, what is the rate of water flow across the capillary wall?

4.50 mL/min Water flow = K f × Net pressure = 0.5 mL/min/mm Hg × 9 mm Hg = 4.5 mL/min

The following measurements were obtained in a male patient: Central venous pressure: 10 mm Hg Heart rate: 70 beats/min Pulmonary vein [O2] = 0.24 mL O2/mL Pulmonary artery [O2] = 0.16 mL O2/mL Whole body O2 consumption: 500 mL/min What is this patient's cardiac output? (A) 1.65 L/min (B) 4.55 L/min (C) 5.00 L/min (D) 6.25 L/min (E) 8.00 L/min

6.25 L/min 500/(0.24-0.16) Fick Equation - don't forget it. =)

36/M placed on ventilator with rate of 16, TV=600mL, FiO2 = 1.0 ABG reveals PO2=350mmHg PCO2 = 36mmHg pH = 7.32 At barometric pressure = 757mmHg, with normal respiratory exchange ratio (R) of 0.8, What is the patient's alveolar oxygen tension?

665mmHg (PAO2=PIO2-(PaCO2/R) = (1.0) (757-47) - (36/0.8) = 710-45 = 665mmHg

Oxygen in the blood is transported two ways?

98% via HgB (as Oxyhemoglobin) 2% freely-dissolved substance

Which of the following causes rigor in skeletal muscle? (A) No action potentials in motoneurons (B) An increase in intracellular Ca2+ level (C) A decrease in intracellular Ca2+ level (D) An increase in adenosine triphosphate (ATP) level (E) A decrease in ATP level

A decrease in ATP level

A man falls into a deep sleep with one arm under his head. This arm is paralyzed when he awakens, but it tingles, and pain sensation in it is still intact. The reason for the loss of motor function without loss of pain sensation is A. A fibers are more susceptible to hypoxia than B fibers. B. A fibers are more sensitive to pressure than C fibers. C. C fibers are more sensitive to pressure than A fibers. D. Motor nerves are more affected by sleep than sensory nerves. E. Sensory nerves are nearer the bone than motor nerves and hence are less affected by pressure.

A fibers are more sensitive to pressure than C fibers. most susceptible to hypoxia: B most susceptible to pressure: A most susceptible to local anesthesia: C

Sensory systems code for the following attributes of a stimulus: A. modality, location, intensity, and duration. B. threshold, receptive field, adaptation, and discrimination. C. touch, taste, hearing, and smell. D. threshold, laterality, sensation, and duration. E. sensitization, discrimination, energy, and projection

A. modality, location, intensity, and duration.

Most potent vasoconstrictor

ADH

Air in the respiratory unit of the lung (respiratory zone) that does NOT undergo gas exchange due to V/Q mismatch normal value?

ALVEOLAR DEAD SPACE Normal Value: 0 mL

Mechanism for DPPC reducing surface tension

AMPHIPATHIC NATURE (HYDROPHOBIC AND HYDROPHILIC)

Air from the Nose to Terminal Bronchioles (conducting zone) that does NOT undergo gas exchange normal value?

ANATOMIC DEAD SPACE normal value: 150 mL

Anomic aphasia is due to damage in what area?

ANGULAR GYRUS Anomic aphasia (also known as dysnomia, nominal aphasia, and amnesic aphasia) is a mild, fluent type of aphasia where individuals have word retrieval failures and cannot express the words they want to say (particularly nouns and verbs)

increases when CVP increases???

ANP

66/M has diastolic murmur over L sternal border, decreased diastolic pressure, increased pulse pressure

AORTIC REGURGITATION If you see murmur with "wide pulse pressure" (or a BP reading na sobrang taas ng systolic pressure and sobrang baba ng diastolic pressure, e.g. BP=150/20), think Aortic Regurgitation.

75/F with exertional dyspnea, and episode of syncope while dancing with her husband. (+) prominent systolic ejection click and crescendo- decrescendo murmur over the R sternal border that radiates to the carotid arteries

AORTIC STENOSIS (WITH DECREASED PULSE PRESSURE)

Contains stressed volume in circulation

ARTERIES

Site of greatest resistance in circulation

ARTERIOLES may rapidly dilate and constrict 𝛂1-Vasoconstricts; 𝜷2-Vasodilates

Macroglia that send processes that envelop synapses and the surface of nerve cells, and helps form the BBB if found in white matter, called? if found in gray matter, called?

ASTROCYTES white matter: FIBROUS ATROCYTES gray matter: PROTOPLASMIC ASTROCYTES - produce substances that are tropic to neurons to help maintain appropriate concentration of ions and NTs by taking up K+ and the NTs Glutamate and GABA

4th heart sound maybe heard in this part of the cardiac cycle due to atria contracting against stiff ventricles (e.g. in LV hypertrophy)

ATRIAL CONTRACTION

among the pacemakers of the heart, which has slowest conduction velocity? fastest? greatest Intrinsic rate of Phase 4 Depolarization (and Heart Rate)?

AV Node slowest conduction velocity at 0.01-0.05 m/sec Purkinje Fibers: 15-20 beats/min fastest conduction velocity at 2-4 m/sec Intrinsic rate of Phase 4 Depolarization (and Heart Rate): SA Node > AV Node > His-Purkinje

generalized nonconvulsive seizures with spike-and-wave discharge in the EEG

Absence seizures

Occurs during an AP when no new AP can be elicited no matter how large the stimulus basis?

Absolute Refractory Period (ARP) Basis: closed Na+-inactivation gates

Occurs when cell membrane is depolarized but not rapidly enough, thus causing Na-inactivation gates to eventually closea -> no AP

Accommodation

Which of the following temporal sequences is correct for excitation- contraction coupling in skeletal muscle? (A)Increased intracellular [Ca2+]; action potential in the muscle membrane; cross-bridge formation (B) Action potential in the muscle membrane; depolarization of the T tubules; release of Ca2+ from the sarcoplasmic reticulum (SR) (C)Action potential in the muscle membrane; splitting of adenosine triphosphate (ATP); binding of Ca2+ to troponin C (D)Release of Ca2+ from the sarcoplasmic reticulum (SR); depolarization of the T tubules; Action potential in the muscle membrane

Action potential in the muscle membrane; depolarization of the T tubules; release of Ca2+ from the sarcoplasmic reticulum (SR)

Blow flow increases to meet increased metabolic demand

Active Hyperemia vs Reactive Hyperemia: ↑ in blood flow in response to brief period of ↓ blood flow

Which autonomic receptor mediates an increase in heart rate? (A) Adrenergic α1 receptors (B) Adrenergic β1 receptors (C) Adrenergic β2 receptors (D) Cholinergic muscarinic receptors (E) Cholinergic nicotinic receptors

Adrenergic β1 receptors Remember: puso at bato, Beta-1.

Which autonomic receptor is activated by low concentrations of epinephrine released from the adrenal medulla and causes vasodilation? (A) Adrenergic α1 receptors (B) Adrenergic β1 receptors (C) Adrenergic β2 receptors (D) Cholinergic muscarinic receptors (E) Cholinergic nicotinic receptors

Adrenergic β2 receptors Remember: Beta-2 will always cause smooth muscle RELAXATION while Alpha-1 will always cause smooth muscle CONTRACTION

BRAIN WAVES that disappear when a patient's eye is open:

Alpha rhythm/waves alpha: awake; at rest, eyes closed A as in "ahhh" relaxed coz eyes closed

FICK EQUATION

Alternative way of computing for cardiac output TANDAAN DAW

A major cause of cor pulmonale in COPD is a decrease in?

Alveolar PO2

A 49-year-old man has a pulmonary embolism that completely blocks blood flow to his left lung. As a result, which of the following will occur? (A) Ventilation/perfusion (V/Q) ratio in the left lung will be zero (B) Systemic arterial PO2 will be elevated (C) V/Q ratio in the left lung will be lower than in the right lung (D) Alveolar PO2 in the left lung will be approximately equal to the PO2 in inspired air (E) Alveolar PO2 in the right lung will be approximately equal to the PO2 in venous blood

Alveolar PO2 in the left lung will be approximately equal to the PO2 in inspired air Because no blood flow → no gas exchange. Alveolar PO2 with therefore be the same as inspired air (hindi pumpunta oxygen from alveoli to blood)

A 38-year-old woman was referred to a psychiatrist after telling her primary care physician that she had difficulty sleeping (awakening at 4 AM frequently for the past few months) and a lack of appetite causing a weight loss of over 20 lb. She also said she no longer enjoyed going out with her friends or doing volunteer service for underprivileged children. What type of drug is her doctor most likely to suggest as an initial step in her therapy? A. A serotonergic receptor antagonist B. An inhibitor of neuronal uptake of serotonin C. An inhibitor of monoamine oxidase D. An amphetamine-like drug E. A drug that causes an increase in both serotonin and dopamine

An inhibitor of neuronal uptake of serotonin The case demonstrated a patient experiencing signs and symptoms of depression. Focus in treating clinical depression has shifted from norepinephrine to serotonin. It is known that the primary serotonin metabolite 5-HIAA is low in CSF of depressed individuals. Drugs such as fluoxetine (Prozac), which inhibit serotonin reuptake without affecting norepinephrine reuptake, are effective as antidepressants.

anemia vs polycythemia - which has turbulent blood flow? higher resistance to blood flow?

Anemia: turbulent blood flow Polythemia: higher resistance to blood flow.

Administration of a lethal dose of carbon monoxide is considered what type of hypoxia ? * A. Hypoxic hypoxia B. Anemic hypoxia C. Ischemic/stagnant hypoxia D. Histotoxic hypoxia E. None of the above

Anemic hypoxia

what lung zone in the ff: Apex of the Lungs (standing) Base of the Lungs (standing) Entire lungs In a Supine Position Entire lungs During exercise Pulmonary Hemorrhage Positive Pressure Ventilation

Apex of the Lungs (standing) - ZONE 2 & 3 Base of the Lungs (standing) - ZONE 3 Entire lungs In a Supine Position - ZONE 3 Entire lungs During exercise - ZONE 3 Pulmonary Hemorrhage - ZONE 1 ☹ Positive Pressure Ventilation - ZONE 1 ☹

Which of the following is not involved in color vision? A. Activation of a pathway that signals differences between S cone responses and the sum of L and M cone responses B. Geniculate layers 3-6 C. P pathway D. Area V3A of visual cortex E. Area V8 of visual cortex

Area V3A of visual cortex Area V3A of visual cortex is responsible for motion and not for color vision.

simple vs facilitated diffusion: which is faster at low concentration? at high concentrations?

At low-solute concentrations: Facilitated Diffusion is faster than Simple diffusion (because FD involves carrier proteins, which makes it go faster, analogous to a person riding a fast car) At high-solute concentrations: Facilitated Diffusion is slower than Simple Diffusion (because FD exhibits saturation and transport maximum (basically a speed limit), unlike SD)

normal cardiac output? max CO in athletes? in non-athletes?

At rest: 5L/min Max CO (Non-Athlete): 20L/min Max CO (Athlete): 30L/min

Clinical Conditions associated with cerebellar dysfunction (4)

Ataxia dysdiadochokinesia Intention Tremor Absent Rebound Phenomenon

Which of the following CNS regions is not correctly paired with a neurotransmitter or a chemical involved in pain modulation? A. Periaqueductal gray matter and morphine B. Nucleus raphe magnus and norepinephrine C. Spinal dorsal horn and enkephalin D. Dorsal root ganglion and opioids E. Spinal dorsal horn and serotonin

B • Nucleus raphe magnus and serotonin • Locus Coeruleus and norepinephrine (iLOCUS NortE) • Periaqueductal gray matter and morphine • Spinal dorsal horn and enkephalin • Dorsal root ganglion and opioids • Spinal dorsal horn and serotonin

Which of the following electrophysiologic events is correctly paired with the change in ionic currents causing the event? A. Fast inhibitory postsynaptic potentials (IPSPs) and closing of Cl- channels B. Fast excitatory postsynaptic potentials (EPSPs) and an increase in Ca2+ conductance C. Endplate potential and an increase in Na+ conductance D. Presynaptic inhibition and closure of voltage-gated K+ channels E. Slow EPSPs and an increase in K+ conductance

B and C Endplate Potential - Na+ Influx Fast IPSP - Cl- Influx - K+ efflux - closure of Na+ or Ca2+ channels Fast EPSP - Na+ Influx - Ca2+ influx Slow EPSP - Slow K+ efflux Presynaptic Inhibition - K+ Efflux

Which of the following receptors is coupled to a heterotrimeric G protein? A. glycine receptor B. GABAB receptor C. nicotinic acetylcholine receptor at myoneural junction D. 5HT3 receptor E. ANP receptor

B. GABAB receptor GABAB receptor: metabotropic/GPCR GABAA receptor: ionotropic (chloride) Nicotinic receptor: ionotropic (sodium/potassium) Glycine receptor: ionotropic (chloride) ANP and 5HT3 receptors: ionotropic

increased venous return leads to increased heart rate and thus increased cardiac output - due to what mechanism?

BAINBRIDGE REFLEX remember: Frank-Starling Mechanism ↑VR → ↑SV → ↑CO Bainbridge Reflex ↑VR → ↑HR → ↑CO

Ohm's Law

BLOOD FLOW Directly proportional to Pressure Difference Inversely proportional to resistance Flow or CO = pressure difference / TPR

Drug that blocks release of Ach from pre-synaptic terminals

BOTULINUS TOXIN Mechanism behind botulinum type A (Botox) smoothing out glabellar lines: Block the release of synaptic transmitter from alpha motoneurons (specifically acetylcholine)

Carbon dioxide (CO2) regulates blood flow to what organ

BRAIN

BUFFER INSIDE RBC? BUFFER IN INTERSTITIAL FLUID?

BUFFER INSIDE RBC IS DEOXYHGB!!! BUFFER IN INTERSTITIAL FLUID IS CARBONIC ACID

What happens to intrapleural pressure and alveolar pressure... before inspiration? during inspiration? during expiration?

Before inspiration: Intrapleural pressure is negative alveolar pressure = 0 During inspiration: Intrapleural pressure becomes more negative alveolar pressure becomes more negative During expiration: Alveolar pressure increases and becomes greater than atmospheric pressure Intrapleural pressure increases back to its resting value ** When you inhale, the contraction of the diaphragm will cause a decrease in intrapleural pressure which will cause a decrease in alveolar pressure. That drop in alveolar pressure will cause the alveolar pressure to be less than atmospheric pressure → this will lead to air moving from the atmosphere and into the alveoli. During expiration, intrapleural pressure will increase which will cause an increase in alveolar pressure. The alveolar pressure will now become greater than atmospheric pressure causing air to move from alveoli to the atmosphere.

predisposes athletes to PVCs?

Bradycardia

3 organs with Autoregulation of Blood Flow

Brain: CO2 and H+ Heart: Perfusion Pressure Kidneys: Tubuloglomerular Feedback

FUNCTIONAL ANATOMIC UNIT OF THE LUNG

Bronchopulmonary Segments (segmental bronchi to alveoli).

7Which of the following is a ligand-gated ion channel? A. VIP receptor B. norepinephrine receptor C. GABAA receptor D. GABAB receptor E. Metamorphic glutamate receptor

C. GABAA receptor GABAA receptor: ionotropic (chloride) GABAB, VIP, norepinephrine, metamorphic glutamate receptor: metabotropic/GPCR

transection at what level of spinal cord will lead to: ____ transection: HR and BP decreases ____ transection: breathing stops ____ transection: death ___: decerebrate rigidity ___: decorticate rigidity

C7 transection: HR and BP decreases C3 transection: breathing stops C1 transection: death decerebrate: above Lateral Vestibular Nucleus between Pontine Reticular Formation and Midbrain decorticate: above Red Nucleus

slowest blood flow velocity in circulation contains ___ and ___ which controls blood flow

CAPILLARIES metarterioles and pre-capillary sphincters control blood flow

Produces protective GAGs and metabolize air-borne toxins

CLARA CELLS / CLUB CELLS

TASTE PATHWAY

CN 7, 9, 10 -> solitary nucleus -> thalamus (VPM nucleus) -> gustatory cortex

Distensibility of the lungs and chest wall

COMPLIANCE Change in volume for a given change in pressure (analogous to capacitance of vessels)

Drug that competes with Ach for receptors on Motor End Plate

CURARE

Repeated stimulation of a skeletal muscle fiber causes a sustained contraction (tetanus). Accumulation of which solute in intracellular fluid is responsible for the tetanus?

Calcium

protein that stores Ca2+ in the SR

Calsequestrin

JOINT & MUSCLE RECEPTORS pulmonary response to Limb Movement

Causes anticipatory increase in Respiratory Rate during Exercise

IRRITANT RECEPTORS in lungs response to noxious stimuli

Causes bronchoconstriction and increases the Respiratory Rate

when stimulated by pulmonary capillary engorgement, J receptors in "juxtacapillary" areas will...?

Causes rapid shallow breathing and responsible for the feeling of dyspnea (e.g. in L-sided heart failure) Responsible for the feelings of dyspnea Here's a mnemonic - J receptors detect what? Jyspnea! =)

central and peripheral chemoreceptors for breathing location? stimulus? action?

Central @ ventral medulla responds to CSF H+ will ↑ RR Peripheral @ Carotid and Aortic Bodies responds to PaO2 <60mmHg (also to high PaCO2, high arterial H+) will ↑ RR

Which autonomic receptor is blocked by hexamethonium at the ganglia, but not at the neuromuscular junction? (A) Adrenergic α1 receptors (B) Adrenergic β1 receptors (C) Adrenergic β2 receptors (D) Cholinergic muscarinic receptors (E) Cholinergic nicotinic receptors

Cholinergic nicotinic receptors

Which autonomic receptor mediates secretion of epinephrine by the adrenal medulla? (A) Adrenergic α1 receptors (B) Adrenergic β1 receptors (C) Adrenergic β2 receptors (D) Cholinergic muscarinic receptors (E) Cholinergic nicotinic receptors

Cholinergic nicotinic receptors

What secretes epinephrine and norepinephrine?

Chromaffin cells in the adrenal medulla - migrated neural cells that secretes Epi and NE

BBB exists in all areas of the brain EXCEPT in ???

Circumventricular Organs (CVOs) • Some areas of the Hypothalamus (including neurohypophysis) • Pineal gland • Area Postrema

Which type of cell in the visual cortex responds best to a moving bar of light? (A) Simple (B) Complex (C) Hypercomplex (D) Bipolar (E) Ganglion

Complex Simple Cells: Bars of light Lang. Complex: gumagalaw na yung bar (Moving Bars). Hypercomplex: yung shape na bar naging Curved na.

In the transport of CO2 from the tissues to the lungs, which of the following occurs in venous blood? (A) Conversion of CO2 and H2O to H+ and HCO3- in the red blood cells (RBCs) (B) Buffering of H+ by oxyhemoglobin (C) Shifting of HCO3- into the RBCs from plasma in exchange for Cl- (D) Binding of HCO3- to hemoglobin (E) Alkalinization of the RBCs

Conversion of CO2 and H2O to H+ and HCO3- in the red blood cells (RBCs)

Which of the following transport processes is involved if transport of glucose from the intestinal lumen into a small intestinal cell is inhibited by abolishing the usual Na+ gradient across the cell membrane? (A) Simple diffusion (B) Facilitated diffusion (C) Primary active transport (D) Cotransport (E) Countertransport

Cotransport SGLT-1 is used to transport glucose from S.I. lumen to S.I. cells. It is a secondary active transport utilizing co-transport-symport. Walang Secondary active transport sa choices, so best answer is co-transport.

Which of the following physiologic processes is not correctly paired with a structure? A. Electrical transmission: gap junction B. Negative feedback inhibition: Renshaw cell C. Synaptic vesicle docking and fusion: presynaptic nerve terminal D. Endplate potential: muscarinic cholinergic receptor E. Action potential generation: initial segment

D The acetylcholine diffuses to the muscle-type nicotinic acetylcholine receptors, which are concentrated at the tops of the junctional folds of the membrane of the motor end plate. Binding of acetylcholine to these receptors increases the Na+ and K+ conductance of the membrane, and the resultant influx of Na+ produces a depolarizing potential, the end plate potential.

A 31-year-old female is a smoker who has had poor oral hygiene for most of her life. In the past few years she has noticed a reduced sensitivity to the flavors in various foods which she used to enjoy eating. Which of the ff is not true about gustatory sensation? A. The sensory nerve fibers from the taste buds on the anterior two-thirds of the tongue travel in the chorda tympani branch of the facial nerve. B. The sensory nerve fibers from the taste buds on the posterior third of the tongue travel in the petrosal branch of the glossopharyngeal nerve. C. The pathway from taste buds on the left side of the tongue is transmitted ipsilaterally to the cerebral cortex. D. Sustentacular cells in the taste buds serve as stem cells to permit growth of new taste buds. E. The pathway from taste receptors includes synapses in the nucleus of the tractus solitarius in the brainstem and ventral posterior medial nucleus in the thalamus.

D The human olfactory epithelium contains 10 to 20 million bipolar olfactory sensory neurons interspersed with glia-like supporting (sustentacular cells) and basal stem cells which permit growth of new taste buds.

Depolarization spreads from sarcolemma to T-Tubules. Then what 2 will be activated? THESE 2 ARE BUY 1 TAKE 1, YOU ACTIVATE ONE AND YOU ACTIVATE THE OTHER! where are they found???

DHPR @ T tubules Ryanodine receptors @ SR (basically calcium channels, so once activated, Ca2+ will go to ICF and bind to troponin C)

Alveolar Macrophages

DUST CELLS

A newly developed local anesthetic blocks Na+ channels in nerves. Which of the following effects on the action potential would it be expected to produce? (A) Decrease the rate of rise of the upstroke of the action potential (B) Shorten the absolute refractory period (C) Abolish the hyperpolarizing afterpotential (D) Increase the Na+ equilibrium potential (E) Decrease the Na+ equilibrium potential

Decrease the rate of rise of the upstroke of the action potential examples: Local anesthetics: lidocaine Class I antiarrhythmic agents Various anticonvulsants: phenytoin, oxcarbazepine (derivative of carbamazepine)

A 53-year-old woman is found, by arteriography, to have 50% narrowing of her left renal artery. What is the expected change in blood flow through the stenotic artery? (A) Decrease to 1⁄2 (B) Decrease to 1⁄4 (C) Decrease to 1⁄8 (D) Decrease to 1⁄16 (E) No change

Decrease to 1⁄16 Radius = 1⁄2 its original value Poiseuille Law Resistance = 8ηl/π 1⁄2)4 = 16x its original value new Q =ΔP/16R The new Q will be 1/16 its original value

An acute decrease in arterial blood pressure elicits which of the following compensatory changes? (A) Decreased firing rate of the carotid sinus nerve (B) Increased parasympathetic outflow to the heart (C) Decreased heart rate (D) Decreased contractility (E) Decreased mean systemic pressure

Decreased firing rate of the carotid sinus nerve

A 24-year-old woman presents to the emergency department with severe diarrhea. When she is supine (lying down), her blood pressure is 90/60 mm Hg (decreased) and her heart rate is 100 beats/min (increased). When she is moved to a standing position, her heart rate further increases to 120 beats/min. Which of the following accounts for the further increase in heart rate upon standing? (A) Decreased total peripheral resistance (B) Increased venoconstriction (C) Increased contractility (D) Increased afterload (E) Decreased venous return

Decreased venous return

A 42-year-old woman with elevated blood pressure, visual disturbances, and vomiting has increased urinary excretion of 3-methoxy-4- hydroxymandelic acid (VMA). A computerized tomographic scan shows an adrenal mass that is consistent with a diagnosis of pheochromocytoma. While awaiting surgery to remove the tumor, she is treated with phenoxybenzamine to lower her blood pressure. What is the mechanism of this action of the drug?

Decreasing IP3/Ca2+ Phenoxybenzamine is an alpha-1 antagonist. Alpha-1 MOA involves increased IP3/Ca2+. Phenoxybenzamine will decrease that. • Best initial treatment for pheochromocytoma: alpha-adrenergic antagonist • Hallmark of pheochromocytoma: HPN

Pulmo function test result in person with pulmonary fibrosis (restrictive lung disease)?

Decreasing diffusing capacity of the lung

Responsible for relaxation of contracted smooth muscles and formation of latch bridges

Dephosphorylation of actomyosin

what happens to Na in depolarization? in repolarization?

Depolarization Opening of Na-Activation Gate (m gate) → Na inward current Repolarization Closure of Na-Inactivation Gate (h gate) → stop Na inward current Opening of K gates → K outward current

In skeletal muscle, which of the following events occurs before depolarization of the T Tubules in the mechanism of excitation- contraction coupling? Depolarization of the sarcolemmal membrane Opening of Ca2+ release channels on the sarcoplasmic reticulum (SR) Uptake of Ca2+ into the SR by Ca2+-adenosine triphosphatase (ATPase) Binding of Ca2+ to troponin C Binding of actin and myosin

Depolarization of the sarcolemmal membrane

A-a GRADIENT - define. Identify if inc or dec in the ff: high altitude hypoventilation diffusion defect V/Q defect right to left shunt

Difference between Alveolar PO2 and Arterial PO2 high altitude: NORMAL hypoventilation: NORMAL diffusion defect: INCREASED V/Q defect: INCREASED right to left shunt: INCREASE ** these five lead to DECREASED PaO2 (hypoxemia)

Potential difference generated across a membrane because of a concentration difference of an ion

Diffusion Potential

main respiratory center supplementary??

Dorsal Respiratory Group (DRG) of Reticular Formation of Medulla - MAIN! for Normal/Resting Inspiration Ventral Respiratory Group (VRG) - Supplements effect of DRG during exercise; for forced inspiration and expiration

Which of the following will double the permeability of a solute in a lipid bilayer? (A) Doubling the molecular radius of the solute (B) Doubling the oil/water partition coefficient of the solute (C) Doubling the thickness of the bilayer (D) Doubling the concentration difference of the solute across the bilayer

Doubling the oil/water partition coefficient of the solute

A 20-year-old woman was diagnosed with Bell palsy (damage to facial nerve). Which of the following symptoms is she likely to exhibit? A. Loss of sense of taste B. Facial twitching C. Droopy eyelid D. Ipsilateral facial paralysis E. All of the above

E The onset of Bell's palsy is fairly abrupt, with maximal weakness being attained by 48 h as a general rule. Pain behind the ear may precede the paralysis for a day or two. Taste sensationmay be l ost unilaterally, and hyperacusis may be present.

epi vs norepi: which has greater B2 effect

EPINEPHRINE Epi has greater Beta-2 effect than NE, kaya siya ginagamit for asthma at hindi NE

Which characteristic or component is shared by skeletal muscle and smooth muscle? (A) Thick and thin filaments arranged in sarcomeres (B) Troponin (C) Elevation of intracellular [Ca2+] for excitation-contraction coupling (D) Spontaneous depolarization of the membrane potential (E) High degree of electrical coupling between cells

Elevation of intracellular [Ca2+] for excitation-contraction coupling Remember this personal mnemonic of mine: Calcium is to muscle what spinach is to Popeye. It provides you with power, the stimulus for contraction.

what causes the following? Endplate Potential (in skeletal muscle motor endplate) Fast Inhibitory Post-Synaptic Potential (IPSP) Fast Excitatory Post-Synaptic Potential (Fast EPSP) Slow Excitatory Post-Synaptic Potential (Slow EPSP) Presynaptic Inhibition

Endplate Potential - Na+ Influx Fast IPSP - Cl- Influx - K+ efflux - closure of Na+ or Ca2+ channels Fast EPSP - Na+ Influx - Ca2+ influx Slow EPSP - Slow K+ efflux Presynaptic Inhibition - K+ Efflux

Diffusion potential that exactly balances (opposes) the tendency for diffusion caused by concentration difference

Equilibrium Potential (Nernst Potential) At electrochemical equilibrium, chemical and electrical driving forces that act on an ion are equal and opposite; no net diffusion occurs ENa+ = +65mV ECa2+ = +120mV EK+ = -85mV ECl- = -85mV

Explicit vs Implicit Memory Working Memory?

Explicit / Declarative Memory: "Knowing what" Implicit Memory: "Knowing How" Working Memory: keeps info available while individual plans actions based on it (e.g. cellphone number)

TRUE OR FALSE: Patients with Transected Spinal Cords have positive nitrogen balance

FALSE! Have negative nitrogen balance as they catabolize large amounts of body protein because they are paralyzed below the level of transection

FAST / FIRST PAIN vs SLOW / SECOND PAIN felt how many seconds after? nerve fibers used? neurotransmitters used?

FAST ✓ after 0.1 sec of stimulus ✓ Type A-delta fibers or Type III ✓ Glutamate * Remember the mnemonic? Just pronounce it really, really fast... glutamate! SLOW ✓ After 1 sec of stimulus ✓ Type C fibers or Type IV ✓ Substance P (relieved by opioids)

Motor, Personality, Calculation, Judgment Somatosensory Cortex Vision Hearing, vestibular processing, recognition of faces, optic pathway (Meyer's Loop)

FRONTAL LOBE PARIETAL LOBE OCCIPITAL LOBE TEMPORAL LOBE

Main Energy for Stroke Work of cardiomyocytes

Fatty Acids

PROPAGATION OF ACTION POTENTIAL: Conduction velocity is increased by what 2 factors

Fiber Size Myelination Remember: the thicker the nerve, the faster it is. The more myelinated the nerve, the faster it is. Conduction Velocity = Distance/Latent Period

Central Auditory Pathways

Fibers ascent through lateral lemniscus to inferior colliculus to medial geniculate nucleus of thalamus to auditory cortex

HypoK ECG findings

Flat/inverted T waves, ↑ amplitude and width of P waves with U waves

increased preload leads to increased stroke volume - due to what mechanism? increased afterload leads to decreased cardiac output - due to what mechanism?

Frank starling mechanism Ohm's law (increased resistance)

equilibrium/resting volume of the lung; marker for lung function; during this time, alveolar pressure = atmospheric pressure

Functional residual capacity

Brain main excitatory neurotransmitter

GLUTAMATE

2 neurotransmitters Involved in Fast Pain and Slow Pain

GLUTAMATE & SUBSTANCE P SP = Substance P = Slow Pain glutamate = pronounce it really really fast!

Produces mucus for lubrication in the respiratory system

GOBLET CELLS

bridge for sharing of small molecules between cells; For rapid intercellular communication found in?? functional unit??

Gap Junctions Cardiac and unitary smooth muscles ** they enable the cells to contract together as one. When you hear gap junctions, think SYNCITIUM - the ability of muscle cells to contract together as one. Functional Unit: ConneXON (its Subunit: ConneXIN)

Which of the following substances crosses capillary walls primarily through water-filled clefts between the endothelial cells? (A) O2 (B) CO2 (C) CO (D) Glucose

Glucose WAG DAW DEDMAHIN THIS Lipid-soluble substances - simple diffusion across capillaries - O2 and CO2 Small water-soluble substances - clefts/pores across capillaries - water, glucose, amino acids Large water-soluble substances - pinocytosis (cellular drinking) - Proteins

Which of the following statements is not true? A. Neuronal glutamate is synthesized in glia by the enzymatic conversion from glutamine and then diffuses into the neuronal terminal where it is sequestered into vesicles until released by an influx of Ca2+ into the cytoplasm after an action potential reaches the nerve terminal. B. After release of serotonin into the synaptic cleft, its actions are terminated by reuptake into the presynaptic nerve terminal, an action that can be blocked by tricyclic antidepressants. C. Norepinephrine is the only small-molecule transmitter that is synthesized in synaptic vesicles instead of being transported into the vesicle after its synthesis. D. Each nicotinic cholinergic receptor is made up of five subunits that form a central channel that, when the receptor is activated, permits the passage of Na+ and other cations. E. GABA transaminase converts glutamate to GABA; the vesicular GABA transporter transports both GABA and glycine into synaptic vesicles.

Glutamate is formed by reductive amination of the Krebs cycle intermediate 𝛼-ketoglutarate in the cytoplasm. Glutamate released into the synaptic cleft is taken up by a Na+-dependent glutamate transporter, and in the astrocyte, it is converted to glutamine. The glutamine enters the neuron and is converted to glutamate.

main form of CO2 transport in blood

HCO3- 70%: HCO3- 23%: CarbaminoHgB 7%: freely-dissolved in plasma BUT THE BUFFER INSIDE RBC IS DEOXYHGB!!! BUFFER IN INTERSTITIAL FLUID IS CARBONIC ACID

Drug that Blocks reuptake of Choline into presynaptic Terminal

HEMICHOLINIUM

Causes of Pulmonary Global Hypoxic Vasoconstriction

HIGH ALTITUDE, FETAL CIRCULATION

Cushing Reaction or Cushing Reflex TRIAD

HPN, Bradycardia, Irregular Respirations

Temperature Regulation Thirst, Food Intake

HYPOTHALAMUS

PROPRIORECEPTORS (determine if fast or slow acting; if type 1 or type 2 receptive field is used) Movement of object on the skin

Hair-end organ

When Detected Temp < Set-Point Temp, this is initiated

Heat Generating Mechanisms Shivering (most potent) Others: Thyroid Hormone production, decreased sweating, piloerection, skin vasoconstriction (𝛼1), brown fat in babies (𝜷3)

When Detected Temp > Set-Point Temperature, this is initiated

Heat Loss Mechanisms Radiation (most potent) Others: convection, Sweating, Skin Vasodilatation, Decreased Heat Production

branch of CN IX that carries signals from carotid sinus to NTS

Hering's nerve

what 2 factors will lead to shift to the LEFT

HgF Carbon Monoxide!!!!

Which of the following does not increase the ability to discriminate many different odors? A. Many different receptors B. Pattern of olfactory receptors activated by a given odorant C. Projection of different mitral cell axons to different parts of the brain D. High β-arrestin content in olfactory neurons E. Sniffing

High β-arrestin content in olfactory neurons Termination of the activity of transducin in photoreception is also accelerated by its binding of β-arrestin." therefore β-arrestin is for vision and not for olfaction. The portion of the nasal cavity containing the olfactory receptors is poorly ventilated in humans, amount of air reaching this region is greatly increased by sniffing. Increased in number of different receptors, pattern of olfactory receptors activated by a given odorant, and projection of different mitral cell axons to different parts of the brain increases the ability to discriminate many different odors

Which of the following statements is correct? A. The motor protein for inner hair cells is prestin. B. The auditory ossicles function as a lever system to convert the resonant vibrations of the tympanic membrane into movements of the stapes against the endolymph-filled scala tympani. C. The loudness of a sound is directly correlated with the amplitude of a sound wave, and pitch is inversely correlated with the frequency of the sound wave. D. Conduction of sound waves to the fluid of the inner ear via the tympanic membrane and the auditory ossicles is called bone conduction. E. High-pitched sounds generate waves that reach maximum height near the base of the cochlea; low-pitched sounds generate waves that peak near the apex.

High-pitched sounds generate waves that reach maximum height near the base of the cochlea; low-pitched sounds generate waves that peak near the apex. • The motor protein for outer hair cells is prestin. • The auditory ossicles function as a lever system to convert the resonant vibrations of the tympanic membrane into movements of the stapes against the endolymph-filled scala media. • The loudness of a sound is directly correlated with the amplitude of a sound wave, and pitch is directly correlated with the frequency of the sound wave. • Bone conduction, is the transmission of vibrations of the bones of the skull to the fluid of the inner ear

A 12-year-old boy has a severe asthmatic attack with wheezing. He experiences rapid breathing and becomes cyanotic. His arterial PO2 is 60 mm Hg and his PCO2 is 30 mm Hg. Which of the following statements about this patient is most likely to be true? (A) Forced expiratory volume/forced vital capacity (FEV1/FVC) is increased (B) Ventilation/perfusion (V/Q) ratio is increased in the affected areas of his lungs (C) His arterial PCO2 is higher than normal because of inadequate gas exchange (D) His arterial PCO2 is lower than normal because hypoxemia is causing him to hyperventilate (E) His residual volume (RV) is decreased

His arterial PCO2 is lower than normal because hypoxemia is causing him to hyperventilate

HYPOXEMIA VS. HYPOXIA

Hypoxemia - Decreased arterial PO2 A-a gradient used hypoxemia Hypoxia - Decreased tissue PO2 Triggers: EPO production (through increased production of hypoxia-inducible factor 1𝝰)

Primary drive to breath in COPD patients?

Hypoxic Drive (low PaO2 stimulating peripheral chemoreceptors. Hypercapneic drive is blunted due to compensated respiratory acidosis)

Effect of Surfactant on Lung Compliance

INCREASE

nitric oxide effect on nerves? vessels?

INHIBITORY neurotransmitter VASODILATOR

the following will lead to changes in what? will affect what part of the heart? INOTROPIC EFFECT LUSITROPIC EFFECT CHRONOTROPIC EFFECT DROMOTROPIC EFFECT

INOTROPIC EFFECT - contractility - affect stroke volume (and CO) LUSITROPIC EFFECT - rate of relaxation CHRONOTROPIC EFFECT - Heart Rate - affect SA node DROMOTROPIC EFFECT - conduction velocity - affect AV node (coz slowest d/t AV nodal delay) - affected by inward Ca channel

phase of cardiac cycle where Ventricular Pressure: rapidly decreases but Ventricular Pressure > Atrial Pressure

ISOVOLUMIC RELAXATION

what phase in cardiac cycle will Incisura / Dicrotic Notch happen?

ISOVOLUMIC RELAXATION slight increase in aortic pressure

channels responsible for slow Na influx during Phase 4 of action potential of pacemaker cells of the heart

If channels or "slow, funny sodium channels" Triggered by K+ efflux of Phase 3 which causes automaticity and pacemaking activity of the SA Node (Phase 3 will always cause Phase 4)

sound in air from outer ear is matched with sound in fluid in inner ear

Impedance matching

A tumor was diagnosed near the base of the skull in a 56-year-old woman, impinging on her optic tract. Which of the following statements about the central visual pathway is correct? A. The fibers from each temporal hemiretina decussate in the optic chiasm, so that the fibers in the optic tracts are those from the temporal half of one retina and the nasal half of the other. B. In the geniculate body, the fibers from the nasal half of one retina and the temporal half of the other synapse on the cells whose axons form the geniculocalcarine tract C. Layers 2 and 3 of the visual cortex contain clusters of cells called globs that contain a high concentration of cytochrome oxidase. D. Complex cells have a preferred orientation of a linear stimulus and, compared to simple cells, are more dependent on the location of the stimulus within the visual field. E. The visual cortex is arranged in horizontal columns that are concerned with orientation.

In the geniculate body, the fibers from the nasal half of one retina and the temporal half of the other synapse on the cells whose axons form the geniculocalcarine tract The visual pathway is from the rods and cones to bipolar cells to ganglion cells then via the optic tract to the thalamic lateral geniculate body to the occipital lobe of the cerebral cortex. The fibers from each nasal hemiretina decussate in the optic chiasm; the fibers from the nasal half of one retina and the temporal half of the other synapse on the cells whose axons form the geniculocalcarine tract.

from non REM to state of awakening, what happens to the following hormones: histamine Ach GABA serotonin norepi

Increase in: Norepinephrine, Serotonin, Histamine Decrease in: Acetylcholine, GABA

Patients are enrolled in trials of a new atropine analog. Which of the following would be expected? (A) Increased AV node conduction velocity (B) Increased gastric acidity (C) Pupillary constriction (D) Sustained erection (E) Increased sweating

Increased AV node conduction velocity Atropine is anti-muscarinic. It will therefore promote mainly sympathetic effects - in this case, choice A. Choice E is not the correct answer since sweating, even if it is sympathetic, utilizes muscarinic receptors as final receptors.

lymph flow is increased by???

Increased capillary permeability others: Massage Negative intrathoracic pressure during inspiration Suction effect of high velocity flow of blood in the veins

When a person moves from a supine position to a standing position, which of the following compensatory changes occurs? (A) Decreased heart rate (B) Increased contractility (C) Decreased total peripheral resistance (TPR) (D) Decreased cardiac output

Increased contractility

Which of the following would occur as a result of the inhibition of Na+,K+- ATPase? (A) Decreased intracellular Na+ concentration (B) Increased intracellular K+ concentration (C) Increased intracellular Ca2+ concentration (D) Increased Na+-glucose cotransport (E) Increased Na+-Ca2+ exchange

Increased intracellular Ca2+ concentration If you inhibit the primary transport Na-K-ATPase pump, you will prevent Na+-gradient in the ECF from occurring. This would then prevent all secondary active transport processes including the Na+-Ca2+ pump (a pump that causes Na+ influx and Ca2+ efflux). Less calcium then goes out, leading to increased intracellular calcium concentration. A drug that inhibits Na+-K+-ATPase pump: think DIGOXIN.

In contraction of gastrointestinal smooth muscle, which of the following events occurs after binding of Ca2+ to calmodulin? (A) Depolarization of the sarcolemmal membrane (B) Ca2+-induced Ca2+ release (C) Increased myosin-light chain kinase (D) Increased intracellular Ca2+ concentration (E) Opening of ligand-gated Ca2+ channels1-

Increased myosin-light chain kinase

Which of the following changes will cause an increase in myocardial O2 consumption? (A) Decreased aortic pressure (B) Decreased heart rate (C) Decreased contractility (D) Increased size of the heart (E) Increased influx of Na+ during the upstroke of the action potential

Increased size of the heart

A 66-year old man with chronic hypertension is treated with prazosin by his physician. The treatment successfully decreases his blood pressure to within the normal range. What is the mechanism of the drug's action? (A) Inhibition of β1 receptors in the sinoatrial (SA) node (B) Inhibition of β2 receptors in the SA node (C) Stimulation of muscarinic receptors in the SA node (D) Stimulation of nicotinic receptors in the SA node (E) Inhibition of α1 receptors on vascular smooth muscle

Inhibition of α1 receptors on vascular smooth muscle Alpha-1 in the blood vessels causes vasoconstriction. Inhibiting alpha-1 can therefore lower BP. Phenoxybenzamine is an alpha-1 antagonist. Alpha-1 MOA involves increased IP3/Ca2+. Phenoxybenzamine will decrease that.

LUNG STRETCH RECEPTORS response to lung distension

Initiates Hering-Breuer Reflex that decreases Respiratory Rate by prolonging expiratory time

which among the starling forces has negative value?

Interstitial Hydrostatic Pressure slightly negative due to lymphatic pump

Which of the following is true during inspiration? (A) Intrapleural pressure is positive (B) The volume in the lungs is less than the functional residual capacity (FRC) (C) Alveolar pressure equals atmospheric pressure (D) Alveolar pressure is higher than atmospheric pressure (E) Intrapleural pressure is more negative than it is during expiration

Intrapleural pressure is more negative than it is during expiration

isometric vs isotonic contraction

Isometric Contraction constant length more tension! pushing against wall Isotonic Contraction constant load more work! (force x distance) dumbbells - concentric contraction - With muscle shortening - eccentric contraction - with muscle lengthening

During which phase of the cardiac cycle is ventricular volume lowest? (A) Atrial systole (B) Isovolumetric ventricular contraction (C) Rapid ventricular ejection (D) Reduced ventricular ejection (E) Isovolumetric ventricular relaxation

Isovolumetric ventricular relaxation

During which phase of the cardiac cycle does the mitral valve open? (A) Atrial systole (B) Isovolumetric ventricular contraction (C) Rapid ventricular ejection (D) Reduced ventricular ejection (E) Isovolumetric ventricular relaxation

Isovolumetric ventricular relaxation The correct answer is actually at the END of isovolumic ventricular relaxation (or the start of rapid ventricular filling)

Depolarization of inner hair cells is caused by:

K+ going into the cells (since endolymph is rich in K+ compared to ICF)

Solutions A and B are separated by a semipermeable membrane that is permeable to K+, but not to Cl-. Solution A is 100 mM KCl, and solution B is 1 mM KCl. Which of the following statements about solution A and solution B is true? (A) K+ ions will diffuse from solution A to solution B until the [K+] of both solutions is 50.5 mM (B) K+ ions will diffuse from solution B to solution A until the [K+] of both solutions is 50.5 mM (C) KCl will diffuse from solution A to solution B until the [KCl] of both solutions is 50.5 mM (D) K+ will diffuse from solution A to solution B until a membrane potential develops with solution A negative with respect to solution B (E) K+ will diffuse from solution A to solution B until a membrane potential develops with solution A positive with respect to solution B

K+ will diffuse from solution A to solution B until a membrane potential develops with solution A negative with respect to solution B K+ is positively charged. It will move from Solution A to B (high concentration to low concentration). negative compared to Solution B.

Blood flow per 100g of tissue is greatest in the ___

KIDNEYS

cell membrane major lipoprotein source of cholesterol

LDL

Causes Bronchoconstriction

LEUKOTRIENES the leukotrienes that cause bronchoconstriction in ASTHMA are your LTC4, D4, E4 - these are collectively called the Slow-Reactive Substance of Anaphylaxis (SRS-A). That's why we use steroids to decrease asthma attacks - to decrease these leukotrienes. Asthma bronchonstriction is due to SRS-A and not due to histamine.

Behavior, Emotions, Motivation

LIMBIC LOBE

vasodilator released by muscles lacking oxygen (it makes sense that it's a vasodilator - to give more blood, oxygen and glucose to the starving muscles)

Lactic Acid can stimulate pain nerve endings - lactic acid is the cause of chest pain in M.I. and muscle pain in muscle fatigue. Lactic Acid is a product of ANAEROBIC glycolysis

Henry's Law for Concentration of Dissolved Gases (Pulmo)

Law for Gases dissolved in solution

Dalton's Law of Partial Pressure (Pulmo)

Law for Mixed Gases

Fick's Law of Diffusion (Pulmo)

Law for transfer of gases through simple diffusion in cell membranes or capillary walls Driving force for diffusion: Partial Pressure difference (NOT concentration of gases)

Boyle's Law (Pulmo)

Law implying thatan ↑in Lung Volume will ↓ Pressure

In pre-term babies, perfect storm - walang mature surfactant kaya mataas ang surface tension; at the same time pre-term nga kaya maliit yung mga alveoli. WHAT LAW IS APPLICABLE HERE

Law of Laplace pressure is directly proportional to surface tension and inversely proportional to radius of alveolus

Common ECF finding in Paradoxical Splitting of the 2nd heart sound (P2 comes before A2) - seen in what disease?

Left Bundle Branch Block

A 60-year-old businessman is evaluated by his physician, who determines that his blood pressure is significantly elevated at 185/130 mm Hg. Laboratory tests reveal an increase in plasma renin activity, plasma aldosterone level, and left renal vein renin level. His right renal vein renin level is decreased. What is the most likely cause of the patient's hypertension? (A) Aldosterone-secreting tumor (B) Adrenal adenoma secreting aldosterone and cortisol (C) Pheochromocytoma (D) Left renal artery stenosis (E) Right renal artery stenosis

Left renal artery stenosis The L renal artery stenosis means less blood in the left kidney, which means less blood pressure in the L kidney. This will stimulate the macula densa in the L kidney, triggering R-A-A-S. This would result in increased BP. The increased aldosterone would cause negative feedback on renin production on the R kidney (the normal kidney) but not on the L kidney (the abnormal kidney) resulting in decreased renin on the R and increased renin on the L.

In which of the following situations is pulmonary blood flow greater than aortic blood flow? (A) Normal adult (B) Fetus (C) Left-to-right ventricular shunt (D) Right-to-left ventricular shunt (E) Right ventricular failure

Left-to-right ventricular shunt

most important characteristic of hydrophobic hormones that governs diffusability across cell membrane

Lipid Solubility

HyperK ECG findings

Low P waves, Tall T waves

Vasomotor Center, Respiratory Center (DRG, VRG), Swallowing, Coughing & Vomiting Centers

MEDULLA

A tissue macrophage that act as scavenger cells, removing debris resulting from injury, infection, and disease (e.g. multiple sclerosis, AIDS-related dementia, Parkinson disease, & Alzheimer disease)

MICROGLIA

67/M with RHD presents with difficulty breathing while exercising. (+) holosystolic murmur at the L 5th ICS MCL. Murmur loudest at the apex, radiates to axilla, enhanced during expiration, and when patient is instructed to make a fist

MITRAL REGURGITATION (WITH INCREASED V WAVE) rheu-M-A-T-ic heart disease valve involvement Mitral > Aortic > Tricuspid

The cross-bridges of the sarcomere in skeletal muscle are made up of?

MYOSIN

INTERCELLULLAR STRUCTURES Disk-shaped; For tight intercellular adhesion found in??

Macula Adherens (Desmosomes) think STAPLER wires! epithelium

Main muscle for normal inspiration? Main muscle for normal expiration? muscle for forced INSPIRATION? muscle for forced EXPIRATION?

Main muscle for normal inspiration: diaphragm Main muscle for normal expiration: none EXTERNAL intercostals are for forced INSPIRATION INTERNAL intercostals are for forced EXPIRATION Wag mapagbaligtad ha. ACCESSORY Muscles are used also for forced inspiration, while ABDOMINAL Muscles are used also for forced expiration

overexcitation of skeletal muscles due to defective ryanodine receptors (that results in excess Ca2+ release from SR) in response to halothane and succinylcholine treatment??

Malignant hyperthermia Treatment: Dantrolene, a Ca2+-channel blocker and muscle relaxant

loss of recent memory is due to damage to what area

Mamillary bodies Cholinergic neurons in the nucleus basalis of Meynert and related areas of the forebrain

Which of the following is the site of highest airway resistance? (A) Trachea (B) Largest bronchi (C) Medium-sized bronchi (D) Smallest bronchi (E) Alveoli

Medium-sized bronchi

PROPRIORECEPTORS (determine if fast or slow acting; if type 1 or type 2 receptive field is used) Movement of objects, low-frequency (slow) vibration, determines texture

Meissner Corpuscles (FA1)

PROPRIORECEPTORS (determine if fast or slow acting; if type 1 or type 2 receptive field is used) 2-point discrimination

Meissner Corpuscles (FA1) Merkel Disc (SA1) both type 1 receptive fields! these 2 sound like GERMAN! they DISCRIMINATE!

steps in photoreception

Memorize all the steps above since it's a favorite in any physio exam. Unique ang vision because hyperpolarization causes the action potential. Somethings to help you: remember, ang Vitamin A, CIS muna bago maging TRANS. Metarhodopsin II activates transducin that activates phosphodiesterase. From here on, negative statements na lahat - DECREASED cGMP, CLOSED Na+ channels, HYPERPOLARIZATION, DECREASED glutamate. Decreased glutamate → Excitatory ionotropic glutamate receptors in bipolar and horizontal cells are inhibited. Decreased glutamate → Inhibitory metabotropic glutamate receptors in bipolar and horizontal cells are excited and depolarized.

PROPRIORECEPTORS (determine if fast or slow acting; if type 1 or type 2 receptive field is used) Gives steady-state signals for continuous touch Localizes touch sensation and to determine texture

Merkel Disc (SA1)

metabotropic vs ionotropic receptors

Metabotropic Receptors: G-protein Coupled Receptors (GPCR) that utilizes second messengers like IP3/DAG or cAMP Ionotropic Receptors: Ion-channel linked-receptors that utilizes ligand-gated ion channels

Which of the following is a step-in photoreception in the rods? (A) Light converts all-trans rhodopsin to 11-cis rhodopsin (B) Metarhodopsin II activates transducin (C) Cyclic guanosine monophosphate (cGMP) levels increase (D) Rods depolarize (E) Release of neurotransmitter increases

Metarhodopsin II activates transducin

PONS

Micturition Center, Pneumotaxic, Apneustic Centers

possible mechanism of sleep is the secretion of ?

Muramyl Peptide

detects muscle length? detects muscle tension? Detect Dynamic changes? Detect Static changes?

Muscle Spindle (Group 1a and 2) "SpindLLLLLLe, Length" 1a - nuclear bag fibers - detect dynamic 2 - nuclear chain - detect static Mnemonic: imagine a girl holding a very expensive bag that a thief tries to steal in broad daylight. Hinampas ni girl si thief using the bag. Yung act ng paghampas - DYNAMIC. Afterwards, dumating yung police, ang they tied up the thief using CHAINS. Hindi makagalaw yung thief - STATIC. --- Golgi Tendon Organs (Group 1b) "Tendon: Tension"

smooth muscle action potential 4 main difference from skeletal/cardiac AP (what 4 components of AP are exclusive to SM)?

Myosin-Light Chain Kinase (MLCK): phosphorylates and activates myosin heads - CONTRACTION! Myosin-Light Chain Phosphatase (MLCP): dephosphorylates and inactivates myosin heads - RELAXATION! Calmodulin: binds with Ca Caldesmon and Calponin: inhibits muscle contraction *** To emphasize: MLCK causes smooth muscle CONTRACTION. MLCP causes smooth muscle RELAXATION. Calmodulin is analogous to Troponin C - it binds with calcium. Caldesmon is analogous to Troponin I - it inhibits actin-myosin interaction.

Drug that Inhibits Acetylcholinesterase

NEOSTIGMINE

At the muscle end plate, acetylcholine (ACh) causes the opening of: Na+ channels and depolarization toward the Na+ equilibrium potential K+ channels and depolarization toward the K+ equilibrium potentialCa2+ channels and depolarization toward the Ca2+ equilibrium potential Na+ and K+ channels and depolarization to a value halfway between the Na+ and K+ equilibrium potentials Na+ and K+ channels and hyperpolarization to a value halfway between the Na+ and K+ equilibrium potentials

Na+ and K+ channels and depolarization to a value halfway between the Na+ and K+ equilibrium potentials

RESTING MEMBRANE POTENTIAL of a nerve (-70mV) is caused by what 3 elements must know, memorize the 3

Nernst Potential for Na+ and K+ K+ Leak Channels Na+-K+-ATPase Pump

Which of the following statements about neurotransmitters is true?A. All neurotransmitters are derived from amino acid precursors. B. Small-molecule neurotransmitters include dopamine, histamine, ATP, glycine, enkephalin, and norepinephrine. C. Large-molecule transmitters include ATP, cannabinoids, substance P, and vasopressin. D. Norepinephrine can act as a neurotransmitter in the periphery and a neuromodulator in the CNS. E. Nitrous oxide is a neurotransmitter in the CNS.

Norepinephrine can act as a neurotransmitter in the periphery and a neuromodulator in the CNS.

normal value of tidal volume divided into??

Normal Value: 500mL o anatomic dead space (150mL) o respiratory unit of the lung (350mL)

only sensation where nerves do not pass though thalamus

OLFACTION

what happens to the following in OBSTRUCTIVE vs RESTRICTIVE lung disease FEV1 FVC FRC

Obstructive Lung Diseases (OLD) have problems with EXPIRATION, typical examples are asthma and COPD. Restrictive Lung Diseases (RLD) have problems with INSPIRATION, typical example is lung fibrosis. In both OLD and RLD, FEV1 and FVC would decrease, but at different rates. In OLD, there is a greater decrease in FEV1 than FVC, while for RLD, there is a greater decrease in FVC rather than FEV1. These would result in a decrease in FEV1/FVC ratio in OLD (since mathematically, pag mas mataas yung pagbagsak ng numerator kaysa sa denominator, bababa yung quotient) and normal or increase FEV1/FVC ratio in RLD (since mathematically, pag mas mataas yung pagbagsak ng denominator kaysa sa numerator, tataas yung quotient). Wag makakalimutan: decreased FEV1/FVC ratio in OLD and increased FEV1/FVC ratio in RLD.

Which of the following characteristics is shared by simple and facilitated diffusion of glucose? (A) Occurs down an electrochemical gradient (B) Is saturable (C) Requires metabolic energy (D) Is inhibited by the presence of galactose (E) Requires a Na+ gradient

Occurs down an electrochemical gradient

cell membrane phospholipids outer vs inner leaflets

Outer Leaflet: C-E-Sphi PhospatidylCholine, PhosphatidylEthanolamine, Sphingomyelin Inner Leaflet: S-I PhosphatidylSerine, PhosphatidylInositol

prostaglandin that ↑set-point temperature in hypothalamus

PGE2: increases set-point temp Mnemonic: PGE1 PGE1: E1 mong bukas ang ductus arteriosus. PGE2: E2 ang para sa fever!

Anatomic + Alveolar Dead Space

PHYSIOLOGIC DEAD SPACE

Pulmonary Circulation vs Systemic Circulation PRESSURE RESISTANCE CARDIAC OUTPUT`

PRESSURE: pulmo < systemic RESISTANCE: pulmo < systemic CO: pulmo = systemic

Estimates Left Atrial Pressure Synonym: Right Atrial Pressure

PULMONARY CAPILLARY WEDGE PRESSURE CENTRAL VENOUS PRESSURE

67/M cardiac transplant candidate has the following labs: Pulmonary Artery Pressure (PAP) = 35mmHg Cardiac Output = 4L/min Left Atrial Pressure(LAP) = 15mmHg Right Atrial Pressure = 10mmHg What is his Pulmonary Vascular Resistance?

PVR = Mean PAP - mean LAP/pulmonary blood flow = 35 - 15mmHg/4L/min = 5mmHg/L/min

PROPRIORECEPTORS (determine if fast or slow acting; if type 1 or type 2 receptive field is used) Detects deep pressure high-frequency (fast) vibration; tapping

Pacinian Corpuscles (FA2) onion-shaped!!!

Asteriognosis and Agnosia is due to damage in what area

Parietal Lobe of the representational hemisphere Asteriognosis - inability to recognize objects by feeling them agnosia - loss of the ability to recognize objects, faces, voices, or places

Degeneration of dopaminergic neurons has been implicated in what disease?

Parkinson's disease

passive vs active tension

Passive Tension: tension due to muscle stretch Active Tension: tension due to muscle contraction; proportional to number of cross-bridge cycles formed

Perfusion-limited Gas Exchange vs Diffusion-limited Gas Exchange

Perfusion-limited: N2O, O2, CO2 under normal conditions - Gas equilibrates with the pulmonary capillary Diffusion-limited: CO and O2 during strenuous exercise and disease states (emphysema, fibrosis) - Gas Does NOT equilibrate Movement of oxygen from alveoli to blood at rest (normal condition) is PERFUSION-LIMITED - meaning mabilis naman yung pag transfer ng gas from alveoli to blood, ang limitation is the number of capillaries and the blood inside it. See the graph of perfusion-limited exchange above - ang bilis maachieve yung peak early sa length ng capillary. Movement of oxygen from alveoli to blood during exercise is DIFFUSION- LIMITED - the limiting factor is the diffusion characteristics of the gas itself, and not blood anymore. In exercise kasi, tumataas yung blood flow to the lungs, so hindi na perfusion-limited ang gas exchange during exercise. Take a look at the graph of diffusion-limited gas exchange above - kahit patapos na ng length ng capillary, hindi pa rin mataas yung peak, kasi nga even with increased blood flow doon, hindi naman nagbabago yung diffusing characteristics ng oxygen itself.

Which person would be expected to have the largest A-a gradient? (A) Person with pulmonary fibrosis (B) Person who is hypoventilating due to morphine overdose (C) Person at 12,000 feet above sea level (D) Person with normal lungs breathing 50% O2 (E) Person with normal lungs breathing 100% O2

Person with pulmonary fibrosis Again, hypoxemia with high A-a gradient: diffusion defect like lung fibrosis, V/Q defect, and R-to-L shunting of the blood.

Bohr equation

Physiologic Dead Space computation

The next normal beat immediately after PVC will have greater force of contraction due to greater Ca influx due to increased ventricular filling time

Postextrasystolic Potentiation (in Premature Ventricular Contraction (PVC) or Extrasystole)

Generates movement plan that is then transferred to primary motor cortex for execution rehearses complex motor sequences Executes movement that is then transferred to brainstem and spinal cord where lower motoneurons causes voluntary movements

Pre-Motor Cortex Supplementary Motor Cortex (BA 6) Primary Motor Cortex (BA 4)

Which of the following is a feature of the sympathetic, but not the parasympathetic, nervous system? (A) Ganglia located in the effector organs (B) Long preganglionic neurons (C) Preganglionic neurons release norepinephrine (D) Preganglionic neurons release acetylcholine (ACh) (E) Preganglionic neurons originate in the thoracolumbar spinal cord

Preganglionic neurons originate in the thoracolumbar spinal cord Sympa will use thoracic nerves and lumbar nerves (thoracolumbar). Para will use cranial nerves and sacral nerves (craniosacral). Remember this mnemonic for the cranial nerves utilized by the parasympathetic nervous system: 1973 (CN X, IX, VII, III). Hindi 1972 (Vit-K dependent clotting factors yun). Hindi rin 1975 (mixed motor-sensory CN yun) Wag malillito sa 1972, 1973, 1975J

A new drug is developed that blocks the transporter for H+ secretion in gastric parietal cells. Which of the following transport processes is being inhibited? (A) Simple diffusion (B) Facilitated diffusion (C) Primary active transport (D) Cotransport (E) Countertransport

Primary active transport H+-K+-ATPase pump is the proper term for the proton pump of the parietal cells of the stomach. It is a Primary Active Transport, countertransport.

where do you expect a Prolonged QT Interval? Shortened QT Interval?

Prolonged QT Interval: HYPOCALCEMIA Shortened QT Interval: HYPERCALCEMIA

Which of the following has a much lower concentration in the cerebrospinal fluid (CSF) than in cerebral capillary blood? (A) Na+ (B) K+ (C) Osmolarity (D) Protein (E) Mg2+

Protein! CSF = blood are ISOOSMOTIC CSF has more Na Blood has more protein

Which of the following statements about glia is true? A. Microglia arise from macrophages outside of the nervous system and are physiologically and embryologically similar to other neural cell types. B. Glia do not undergo proliferation. C. Protoplasmic astrocytes produce substances that are tropic to neurons to help maintain the appropriate concentration of ions and neurotransmitters by taking up K+ and the neurotransmitters glutamate and GABA. D. Oligodendrocytes and Schwann cells are involved in myelin formation around axons in the peripheral and central nervous systems, respectively. E. Macroglia are scavenger cells that resemble tissue macrophages and remove debris resulting from injury, infection, and disease.

Protoplasmic astrocytes produce substances that are tropic to neurons to help maintain the appropriate concentration of ions and neurotransmitters by taking up K+ and the neurotransmitters glutamate and GABA. Microglia arise from macrophages outside of the nervous system and are physiologically and embryologically unrelated to other neural cell types; they are scavenger cells that resemble tissue macrophages and remove debris resulting from injury, infection, and disease." "Oligodendrocytes and Schwann cells are involved in myelin formation around axons in the central and peripheral nervous systems, respectively. Glia undergo proliferation."

3rd Heart sound may be heard in this part of the cardiac cycle

RAPID VENTRICULAR FILLING

phase of cardiac cycle where Ventricular Pressure: rapidly decreases to a point that it is now less than Atrial Pressure

RAPID VENTRICULAR FILLING

Longest phase of the cardiac cycle

REDUCED VENTRICULAR FILLING (DIASTASIS) dependent on HR!!

Vitamin A is a precursor for the synthesis of? rods and cones. retinal. rod transducin. opsin. cone transducin

RETINAL aka Retinene1

When compared with the cones of the retina, the rods (A) are more sensitive to low-intensity light (B) adapt to darkness before the cones (C) are most highly concentrated on the fovea (D) are primarily involved in color vision

RODS are more sensitive to low-intensity light Cones and Rods literally have shapes that look like cones and rods. Cones - think COLOR (DAYTIME) VISION. Rods - think BLACK AND WHITE (NIGHTIME) VISION. Vitamin A deficiency may cause nyctalopia (night blindness) that affects the RODS.

Most potent Heat Loss Mechanism

Radiation (most potent) Others: convection, Sweating, Skin Vasodilatation, Decreased Heat Production

↑ in blood flow in response to brief period of ↓ blood flow

Reactive Hyperemia vs Active Hyperemia: Blow flow increases to meet increased metabolic demand

During which phase of the cardiac cycle is aortic pressure highest? (A) Atrial systole (B) Isovolumetric ventricular contraction (C) Rapid ventricular ejection (D) Reduced ventricular ejection (E) Isovolumetric ventricular relaxation

Reduced ventricular ejection The correct answer is actually BETWEEN rapid ventricular ejection and reduced ventricular ejection.

Occurs during an AP after ARP when a new AP can be elicited by required greater than usual Na+ inward current

Relative Refractory Period (RRP) Basis: prolonged opening of K+ channels

At which site is systolic blood pressure the highest? (A) Aorta (B) Central vein (C) Pulmonary artery (D) Right atrium (E) Renal artery

Renal artery Bakit hindi aorta eh aorta pinakamalapit sa L ventricle? The reason for this → when blood moves from aorta to a branch of that aorta, the change in direction of blood will hit the branching points and increase the pressure slightly. Kaya yung branch point ng aorta (among the choices above, only renal artery is a branch of the aorta) sa renal artery mas mataas ang pressure nyan compared sa aorta itself. Analogy: imagine driving a very fast car along EDSA. Then you made a sudden left turn in one of the side streets and hit the gutter. Mataas ang pressure ngayon dun sa branch points ng EDSA papunta sa side street. Message me on Viber if you have a hard time with this so I can explain better.

RESPIRATORY UNIT OF THE LUNG

Respiratory bronchiole, alveolar ducts, alveolar sacs These are the only 3 areas in the respiratory system capable of gas exchange!!!!!!

lowest pressure in the heart

Right Atrium - around 0-4mmHg "Blood and air will always flow from high pressure to low". Remember that dictum. So, if you want blood to flow from arteries to capillaries to veins, saan pinakamababa ang pressure? Answer: sa veins. All veins will eventually drain into the vena cava and then to the right atrium.

Which of the following causes of hypoxia is characterized by a decreased arterial PO2 and an increased A-a gradient? (A) Hypoventilation (B) Right-to-left cardiac shunt (C) Anemia (D) Carbon monoxide poisoning (E) Ascent to high altitude

Right-to-left cardiac shunt

Which of the following would produce maximum excitation of the hair cells in the right horizontal semicircular canal? (A) Hyperpolarization of the hair cells (B) Bending the stereocilia away from the kinocilia (C) Rapid ascent in an elevator (D) Rotating the head to the right

Rotating the head to the right where you head turns, is where depolarization happens. In the opposite direction, hyperpolarization happens. You turn your head to the right, and the stereocilia bends towards the kinocilium (single long cilium of hair cell), and depolarization happens on the right ear.

PROPRIORECEPTORS (determine if fast or slow acting; if type 1 or type 2 receptive field is used) Heavy and prolonged touch (detects sustained or STEADY PRESSURE) and to signal degree of joint rotation

Ruffini Corpuscles (SA2)

A 28-year-old man was seen by a neurologist because he had experienced prolonged episodes of tingling and numbness in his right arm. He underwent a neurologic exam to evaluate his sensory nervous system. Which of the following receptors is correctly paired with the type of stimulus to which it is most apt to respond? A. Pacinian corpuscle and motion. B. Meissner corpuscle and deep pressure. C. Merkel cells and warmth. D. Ruffini corpuscles and sustained pressure. E. Muscle spindle and tension.

Ruffini corpuscles and sustained pressure.

properties of Carrier-Mediated Transport (3)

S-S-C - stereospecificity, saturation and competition ay shape lang na pwedeng gumamit ng carrier protein (stereospecificity), limited and number ng carrier protein (saturation), at pwedeng magkaroon ng competition for the carrier proteins (competition: parang "Trip to Jerusalem").

what phase in cardiac cycle will S1 occur? S2? S3? S4?

S1 - isovolumic contraction S2 - isovolumic relaxation - S3 - Rapid Ventricular Filling Normal in children, young adults, pregnant patients; L-Sided S3 in patients with CHF indicative of CV morbidity, mortality S4 - Atrial Contraction/ Systole Stiff Ventricles/Vibration in the ventricular wall during systole

SGLT 1 and 2 location

SGLT-1: SI SGLT-2: Kidneys Think of it this way: you have 1 Intestine, but you have 2 kidneys: SGLT-1 and SGLT-2.

shunt vs dead space

SHUNT= airway obstruction/R to L shunt (V/Q=0) DEAD SPACE= blood flow obstruction (V/Q=infinity)

transport of Oxygen, Nitrogen, CO2 across the cell is via? others that move via this?

SIMPLE DIFFUSION alcohol, anesthetic drugs, lipid hormones (STEROID HORMONES aka aldosterone, sex hormones)

Blood flow to which organ is controlled primarily by the sympathetic nervous system rather than by local metabolites?

SKIN

The ventricles are completely depolarized during which isoelectric portion of the electrocardiogram (ECG)? (A) PR interval (B) QRS complex (C) QT interval (D) ST segment (E) T wave

ST segment - corresponds to phase 2 (pla2/plateau) of cardiac action potential SEGMENT = ISOELECTRIC! think of ZERO! 0 mV!

Most important determinant of Pulse Pressure

STROKE VOLUME PULSE PRESSURE = Systolic Pressure - Diastolic Pressure = Stroke Volume / Arterial Compliance **So, what happens to pulse pressure as you grow older? As you grow older, arterial compliance decreases (arteries become stiffer) due to arteriolosclerosis. Because arterial compliance decreases as you grow older, Stroke Volume INCREASES as you grow older. These are the kinds of questions that you may encounter in your med boards - it's a favorite din kasi in med school.

Pulmonary Blood Flow: SUPINE Pulmonary BLOOD FLOW: STANDING

SUPINE: SAME THROUGH THE ENTIRE LUNG STANDING: LOWEST AT THE APEX, HIGHEST AT THE BASE

normal value of stroke volume? of cardiac output??? of ejection fraction? GIVE FORMULA OF EACH

SV = EDV - ESV = 70mL CO = HR x SV = 5L/min EF = SV/EDV = 55%

sweat glands, piloerector muscles - sympa or para? what receptors are used at postganglionic site?

SYMPATHETIC but uses Ach receptors - MUSCARINIC

normal blood pressure at: SYSTEMIC large arteries arterioles capillaries vena cava PULMONARY pulmonary artery capillaries

SYSTEMIC large arteries: 120/80 mmHg arterioles: 50 mmHg capillaries: 17 mmHg vena cava: 0-4 mmHg PULMONARY pulmonary artery: 25/8 mmHg capillaries: 7 mmHg

myelin is produced by??

Schwann Cells (in the peripheral nervous system or PNS) Oligodendrocytes (in the central nervous system or CNS)

DOPAMINE secreted in? increased in what disease? decreased in?

Secreted in the substantia nigra (fine-tunes movement) ↓ in Parkinson Disease ↑ D2 in Schizophrenia

VESTIBULAR SYSTEM detect angular acceleration? detect horizontal (linear) acceleration? detect vertical (linear) acceleration?

Semicircular Canals: detect angular acceleration Detects position of head in space: Otolith organs: § Utricle: detect horizontal (linear) acceleration § Saccule: detect vertical (linear) acceleration

A 55-year-old woman had been receiving long-term treatment with phenelzine for her depression. After she consumed Chianti wine, aged cheddar cheese, processed meats, and dried fruits one night at a party, the following symptoms developed: a severe headache, chest pain, rapid heartbeat, enlarged pupils, increased sensitivity to light, and nausea. What is the most likely cause of these symptoms? A. The foods were contaminated with botulinum toxin. B. She had a myocardial infarction. C. She experienced a migraine headache. D. She had an adverse reaction to the mixture of alcohol with her antidepressant. E. She had a hypertensive crisis from eating foods high in tyramine while taking a monoamine oxidase inhibitor for her depression.

She had a hypertensive crisis from eating foods high in tyramine while taking a monoamine oxidase inhibitor for her depression. Patients taking MAO inhibitors should avoid tyramine-containing foods (aged cheese, cured meats, and pickled food). Tyramine's spectrum of action is similar to that of norepinephrine. In patients treated with MAO inhibitors—particularly inhibitors of the MAO-A isoform—this effect of tyramine may be greatly intensified, leading to marked increases in blood pressure. This occurs because of increased bioavailability of tyramine and increased neuronal stores of catecholamines.

A 20-year-old female student awakens one morning with severe pain and blurry vision in her left eye; the symptoms abate over several days. About 6 months later, on a morning after playing volleyball with friends, she notices weakness but not pain in her right leg; the symptoms intensify while taking a hot shower. Which of the following is most likely to be the case? A. The two episodes described are not likely to be related. B. She may have primary-progressive multiple sclerosis. C. She may have relapsing-remitting multiple sclerosis. D. She may have a lumbar disk rupture. E. She may have Guillain-Barre syndrome.

She may have relapsing-remitting multiple sclerosis. Typical physiological deficits of multiple sclerosis range from muscle weakness, fatigue, diminished coordination, slurred speech, blurred or hazy vision, bladder dysfunction, and sensory disturbances. Symptoms are often exasperated by increased body temperature or ambient temperature. In the most common form, transient episodes appear suddenly, last a few weeks or months, and then gradually disappear. Subsequent episodes can appear years later, and eventually full recovery does not occur. Others have a progressive form of the disease in which there are no periods of remission.

Most potent Heat-Generating Mechanism

Shivering (most potent) Others: Thyroid Hormone production, decreased sweating, piloerection, skin vasoconstriction (𝛼1), brown fat in babies (𝜷3)

32/M severe respiratory disease after aspiration pneumonia. Inhaled NO given, and patient placed in prone position. Mean pulmonary capillary oxygen content = 19mL/dL Arterial O2 content = 18 mL/dL Mixed Venous O2 content = 14 mL/dL Cardiac Output = 6L/min What is the patient's shunt fraction (ratio of shunted to total pulmonary blood flow)?

Shunt Fraction = (CCO2 - Ca2)/(CCO2-CvO2) =(19mL/dL-18mL/dL)/(19mL/dL-14mL/dL) = 0.2

Transport of D- and L-glucose proceeds at the same rate down an electrochemical gradient by which of the following processes? (A) Simple diffusion (B) Facilitated diffusion (C) Primary active transport (D) Cotransport (E) Countertransport

Simple diffusion Glucose is supposed to be transported via carrier-mediated means. Dapat may stereospecificity, so normally hindi same rate and transport ng D-glucose at L-glucose. Pag same lang rate of transport ng D-glucose and L-glucose, ibig sabihin hindi siya carrier-mediated - simple diffusion na siya. Ingat, this is a tricky question since the transport of D-glucose and L-glucose is not through simple diffusion in real life.

intra or extracellular Ca?? Skeletal Muscle: relies purely on ___ Smooth Muscle: relies mainly on ___ Cardiac Muscle: relies on __

Skeletal Muscle: relies purely on intracellular Ca2+ (SR Ca2+) Smooth Muscle: relies mainly on extracellular Ca2+ (ECF Ca2+) Cardiac Muscle: relies on both intracellular and extracellular Ca2+

what channels are responsible for After-hyperpolarization?

Slow closure of voltage-gated K+ channels

for continuous stimulus strength (detects steady stimulus) for detecting change in stimulus strength (detects onset and offset of stimulus)

Slowly-adapting Receptors (Tonic Receptors) Muscle Spindle, Golgi Tendon, Baroreceptor, Chemoreceptor, Pressure, Slow Pain Receptor Rapidly-adapting Receptors (Phasic Receptors) Pacinian Corpuscle, Light Touch -- When sensory receptors "adapt", they become non-responsive. So youhave two types of receptors - "martyrs" and "players". Slowly-adaptingreceptors are "martyrs" - as long as you love them, kahit kaunti lang, theywill love you back. As long as may stimulus, may response pa rin yungsensory receptor. Matagal sila mag "adapt"; matagal sila maging manhid. Rapidly-adapting receptors are "players" - ang dali nila magsawa, angdali nila maging "manhid." Kahit mahal mo pa sila, hindi ka na nila mahal.Kahit may stimulus pa, wala nang response yung sensory receptors. They only enjoy the start and end of relationships - they only detect the onsetand offset of stimulus.

Solutions A and B are separated by a semi-permeable membrane. Solution A contains 1 mM sucrose and 1 mM urea. Solution B contains 1 mM sucrose. The reflection coefficient for sucrose is one and the reflection coefficient for urea is zero. Which of the following statements about these solutions is correct? (A) Solution A has a higher effective osmotic pressure than solution B (B) Solution A has a lower effective osmotic pressure than solution B (C) Solutions A and B are isosmotic (D) Solution A is hyperosmotic with respect to solution B, and the solutions are isotonic (E) Solution A is hyposmotic with respect to solution B, and the solutions are isotonic

Solution A is hyposmotic with respect to solution B, and the solutions are isotonic Iba ang osmotic pressure sa effective osmotic pressure. Urea osmotic pressure is 1mM. Pero yung effective osmotic pressure niya is zero. So, solution is will be hyperosmotic but same lang ang tonicity compared sa A (since ang terms na "isotonic", "hypertonic" and "hypotonic" refer to EFFECTIVE osmotic pressure and not just osmotic pressure)

A 45-year-old woman visited her clinician after experiencing sudden onset of vertigo, tinnitus & hearing loss in her left ear, nausea, and vomiting. This was the second episode in the past few months. She was referred to an otolaryngologist to rule out Ménière disease. Which of the following statements correctly describe the functions of the external, middle, or inner ear? A. Sound waves are funneled through the external ear to the external auditory meatus and then they pass inward to the tympanic membrane. B. The cochlea of the inner ear contains receptors for hearing, semicircular canals contain receptors that respond to head tilt, and the otolith organs contain receptors that respond to rotation. C. Contraction of the tensor tympani and stapedius muscles of the middle ear cause the manubrium of the malleus to be pulled outward and the footplate of the stapes to be pulled inward. D. Sound waves are transformed by the eardrum & auditory ossicles into movements of the footplate of the malleus. E. The semicircular canals, the utricle, and the saccule of the middle ear are concerned with equilibrium.

Sound waves are funneled through the external ear to the external auditory meatus and then they pass inward to the tympanic membrane. The cochlea of the inner ear contains receptors for hearing, otolith organs contain receptors that respond to head tilt, and the semicircular canals contain receptors that respond to rotation. Contraction of the tensor tympani and stapedius muscles of the middle ear cause the manubrium of the malleus to be pulled inward and the footplate of the stapes to be pulled outward. Sound waves are transformed by the eardrum and auditory ossicles into movements of the footplate of the stapes. The semicircular canals, the utricle, and the saccule of the inner ear are concerned with equilibrium

in exercise among cardiac transplant patients, cardiac output increases mainly due to increase in?

Stroke Volume

Mediates synaptic transmission between pain fibers from pelvis and spinal cord in a patient with gonorrhea

Substance P

Which of the following is not correctly paired? A. Synaptic transmission: Antidromic conduction B. Molecular motors: Dynein and kinesin C. Fast axonal transport: ~400 mm/day D. Slow axonal transport: 0.5-10 mm/day E. Nerve growth factor: Retrograde transport

Synaptic transmission: Antidromic conduction - ORTHODROMIC DAPAT! In the natural situation, impulses pass in one direction only, ie, from synaptic junctions or receptors along axons to their termination. Such conduction is called ORTHODROMIC. Conduction in the opposite direction is called antidromic. Because synapses, unlike axons, permit conduction in one direction only, an antidromic impulse will fail to pass the first synapse they encounter and die out at that point.

troponin T, I, C

T for tropomyosin (Troponin T is found in tropomyosin) I for inhibition (Troponin I inhibits actin-myosin interaction C for calcium (Troponin C is the one that binds with calcium)

invaginations of the sarcolemma; spreads the action potential to all parts of the muscles; contains DHPR

T-Tubules when discussing inVAGINAtion, think TT(ubules)

Relay Center for almost all sensations, Memory Recall

THALAMUS

41/M IV drug user has early systolic murmur. Distance between the height of the blood in the R IJV and sternal angle is 7cm (normal is 3cm)

TRICUSPID REGURGITATION

Other Lung Vasoactive Substances

TXA2, PGI2

TYPE I vs TYPE 2 PNEUMOCYTE

TYPE I PNEUMOCYTE For gas exchange; LARGER! TYPE II PNEUMOCYTE For surfactant production smol lang

As a result of an automobile accident, a 10-year-old boy suffered damage to the brain including the periamygdaloid, piriform, and entorhinal cortices. Which of the following sensory deficits is he most likely to experience? A. Visual disturbance B. Hyperosmia C. Auditory problems D. Taste and odor abnormalities E. No major sensory deficits

Taste and odor abnormalities The pathway to the amygdala is probably involved with the emotional responses to olfactory stimuli, and the pathway to the entorhinal cortex is concerned with olfactory memories. Damage along this pathway would result to odor and taste abnormalities.

A 42-year-old woman with severe pulmonary fibrosis is evaluated by her physician and has the following arterial blood gases: pH = 7.48, PaO2 = 55 mm Hg, and PaCO2 = 32 mm Hg. Which statement best explains the observed value of PaCO2? (A) The increased pH stimulates breathing via peripheral chemoreceptors (B) The increased pH stimulates breathing via central chemoreceptors (C) The decreased PaO2 inhibits breathing via peripheral chemoreceptors (D) The decreased PaO2 stimulates breathing via peripheral chemoreceptors (E) The decreased PaO2 stimulates breathing via central chemoreceptors

The decreased PaO2 stimulates breathing via peripheral chemoreceptors -- Central @ ventral medulla responds to CSF H+ will ↑ RR Peripheral @ Carotid and Aortic Bodies responds to PaO2 <60mmHg (also to high PaCO2, high arterial H+) will ↑ RR

A patient enters the hospital for evaluation of deafness. He is found to also have an elevated plasma renin, although his blood pressure is 118/75 mm Hg. Mutation of what single gene may explain these findings? A. The gene for barttin B. The gene for Na+ channel C. The gene for renin D. The gene for cystic fibrosis transmembrane conductance regulator E. The gene for tyrosine hydroxylase

The gene for barttin Bartter syndrome is due to defective transport in the thick ascending limb. It is characterized by chronic Na+ loss in the urine, with resultant hypovolemia causing stimulation of rennin and aldosterone secretion. The condition can be caused by loss-of function mutations in the gene for any of four key proteins: the Na-K-2Cl cotransporter, the ROMK K+ channel,the ClC-Kb Cl- channel, or barttin

Which of the following statements about the olfactory system is true? (A) The receptor cells are neurons (B) The receptor cells are sloughed off and are not replaced (C) Axons of cranial nerve (CN) I are A-delta fibers (D) Axons from receptor cells synapse in the prepiriform cortex (E) Fractures of the cribriform plate can cause inability to detect ammonia

The receptor cells are neurons OLFACTORY NERVE 1. Only Neurons capable of reproduction 2. uses Type C nerve fibers (the slowest, least precise and accurate nerve fiber) - CN5 3. it is the only sensory modality that does NOT send fibers to the thalamus

thick vs thin filaments (contains what?)

Thick Filament: contains myosin that act as cross- bridges of the sarcomeres Thin Filament: contains actin, tropomyosin, troponin

____: binds myosin to Z lines, binds Z lines to M line (determines normal stiffness of the ventricular muscle) ____: stabilizes sarcolemma and prevents contraction- induced rupture (binds actin to beta-dystroglycan in the sarcolemma) ____: binds Actin to Z lines ____: binds Z lines to sarcolemma ____: acts as molecular rulers that sets the length of actin

Titin: binds myosin to Z lines, binds Z lines to M line (determines normal stiffness of the ventricular muscle) Dystrophin: stabilizes sarcolemma and prevents contraction- induced rupture (binds actin to beta-dystroglycan in the sarcolemma) Actinin, CapZ Protein: binds Actin to Z lines Desmin: binds Z lines to sarcolemma Nebulin: acts as molecular rulers that sets the length of actin

Which of the following parameters is decreased during moderate exercise? (A) Arteriovenous O2 difference (B) Heart rate (C) Cardiac output (D) Pulse pressure (E) Total peripheral resistance (TPR)

Total peripheral resistance (TPR)

Transcellular Transport vs Paracellular Transport

Transcellular Transport: movement across apical and basolateral sides Paracellular Transport: movement through TIGHT JUNCTIONS (aka zonula occludens)

continued degeneration of the dopaminergic neurons of the Substantia Nigra will lead to?

Tremors, cogwheel Rigidity, reduced voluntary movement (Akinesia), Postural problems (Mnemonic: "T-R-A-P" of Parkinson Disease)

"relaxing protein" that covers actin binding sites at rest

Tropomyosin

what kind of nerve fibers do your proprioceptors use?

Type A alpha Ia: muscle spindle Ib: golgi tendon organs - TYPE A (Type I): Thicker, More Myelinated, Faster For temporal and spatial fidelity TYPE C (Type IV): Thinner, Unmyelinated, Slower Less Energy-requiring

smooth muscle: 2 kinds of muscles

Unitary smooth muscle: (+) gap junctions, (+) syncytium, for gross motor movements e.g. uterus aka SINGLE-UNIT / UNITARY SMOOTH MUSCLE / SYNCITIAL SMOOTH MUSCLE / VISCERAL SMOOTH MUSCLE Multi-unit smooth muscle: (-) gap junctions, for fine motor movements e.g. smooth muscle on pupillary constriction!

Which of the following is the result of an inward Na+ current? (A) Upstroke of the action potential in the sinoatrial (SA) node (B) Upstroke of the action potential in Purkinje fibers (C) Plateau of the action potential in ventricular muscle (D) Repolarization of the action potential in ventricular muscle (E) Repolarization of the action potential in the SA node

Upstroke of the action potential in Purkinje fibers

A healthy 65-year-old man with a tidal volume (TV) of 0.45 L has a breathing frequency of 16 breaths/min. His arterial PCO2 is 41 mm Hg, and the PCO2 of his expired air is 35 mm Hg. What is his alveolar ventilation?

VA = (VT-VD) x RR VD = VT x (PaCO2 x PeCO2)/PaCO2 = (0.45) x (41-35)/41 =0.066L VA = (0.45 - 0.066L) x 16 = 6.14L/min

Effect of Hypoxia (low PAO2) on Pulmonary Arterioles

VASOCONSTRICTION

reservoir of blood, where 64% of blood is found has greater ____, ____, and ____

VEINS greater capacitance, distensibility, compliance than arteries

what part of brain is responsible for for control of balance and eye movements? for planning and initiation of movement? for synergy (controls rate, force, range, direction of movement)?

Vestibulocerebellum: for control of balance and eye movements Pontocerebellum: for planning and initiation of movement Spinocerebellum: for synergy (controls rate, force, range, direction of movement)

Main component of Surfactant Active component of Surfactant

WATER DIPALMITOYLPHOSPHATIDYLCHOLINE (DPPC)

What happens to the ff in high altitude Alveolar PO2 Arterial PO2 Pulmonary Vascular Resistance (Hypoxic vasoconstriction) Respiratory Rate Arterial pH HgB Concentration 2,3 BPG

WILL INCREASE: Pulmonary Vascular Resistance (Hypoxic vasoconstriction) Respiratory Rate Arterial pH HgB Concentration 2,3 BPG → shift to the RIGHT WILL DECREASE: Alveolar PO2 Arterial PO2

Wernicke vs Broca aphasia

Wernicke's aphasia: receptive aphasia - "can say, but can't understand" (hence aka fluent aphasia) (Mnemonic: "Wordy Wernicke") For example, they'd say "pangit si doc banzuela" but they dont know what they're talking about! Broca's aphasia - expressive aphasia - "can understand, but cannot say" (Mnemonic: "Broken <speech> Broca") BROCA: frontal lobe

INTERCELLULLAR STRUCTURES • Ring-shaped • increases surface area for contact found in?

Zonula Adherens • Epithelial & endothelial cells • Intercalated disks of cardiac muscles Equivalent in cardiomyocytes is Fascia Adherens (ribbon-like patterns; doesn't completely enclose cell)

Barrier to movement of proteins across membranes; divides cell into apical and basolateral side

Zonula Occludens (Tight Junctions) Leaky: PCT, Jejunum Tight: CD, terminal Colon, BBB

Compared with the apex of the lung, the base of the lung has (A) a higher pulmonary capillary PO2 (B) a higher pulmonary capillary PCO2 (C) a higher ventilation/perfusion (V/Q) ratio (D) the same V/Q ratio

a higher pulmonary capillary PCO2 BASE has higher V higher Q APEX has higher V/Q so better exchange of gases here! More O2, less CO2 :)

Following a sympathectomy, a 66-year-old man experiences orthostatic hypotension. The explanation for this occurrence is (A) an exaggerated response of the renin-angiotensin-aldosterone system (B) a suppressed response of the renin-angiotensin-aldosterone system (C) an exaggerated response of the baroreceptor mechanism (D) a suppressed response of the baroreceptor mechanism

a suppressed response of the baroreceptor mechanism BARORECEPTOR REFLEX - 1st line of defense coz fast! RAAS will only take effect after 20mins

A 50-year-old woman undergoes a neurologic exam that indicates loss of pain and temperature sensitivity, vibratory sense, and proprioception in the left leg. These symptoms could be explained by A. a tumor on the right medial lemniscal pathway in the sacral spinal cord. B. a peripheral neuropathy. C. a tumor on the left medial lemniscal pathway in the sacral spinal cord. D. a tumor affecting the right posterior paracentral gyrus. E. a large tumor in the right lumbar ventrolateral spinal cord.

a tumor affecting the right posterior paracentral gyrus. "The dorsal column-medial lemniscal system carries signals upward to the medulla of the brain and cross to the opposite side in the medulla, they continue through the brain stem to the thalamus by way of the medial lemniscus." "Conversely, signals in the anterolateral system, immediately after entering the spinal cord, synapse in the dorsal horns of the spinal gray matter, then cross to the opposite side of the cord and ascend through the anterior and lateral white columns of the cord. They terminate at all levels of the lower brain stem and in the thalamus." "Thalamic sensory relay area, the third-order nerve fibers project mainly to the postcentral gyrus of the cerebral cortex, which is called somatic sensory area I. "

what phase in cardiac cycle will the ff happen: a wave c wave v wave

a wave: ATRIAL CONTRACTION c wave: ISOVOLUMIC CONTRACTION v wave: ISOVOLUMIC RELAXATION ** venous blood going back to atrium

increases when light strikes the eye?

activity of transducin

A 45-year-old man with testicular cancer underwent chemotherapy treatment with cisplatin. He reported several adverse side effects including changes in taste, numbness and tingling in his fingertips, and reduced sound clarity. When the damage to the outer hair cells is greater than the damage to the inner hair cells A. perception of vertical acceleration is disrupted. B. K+ concentration in endolymph is decreased. C. K+ concentration in perilymph is decreased. D. there is severe hearing loss. E. affected hair cells fail to shorten when exposed to sound

affected hair cells fail to shorten when exposed to sound The outer hair cells respond to sound, like the inner hair cells, but depolarization makes them shorten and hyperpolarization makes them lengthen. This action somehow increases the amplitude and clarity of sounds

what cells have resting membrane potential? what cells have action potential?

all cells have a Resting Membrane Potential But only excitable cells have an Action Potential (neurons, skeletal muscle, cardiac muscles, and smooth muscles)

alpha 1 will always cause ____ beta 2 will always cause ____

alpha 1 will always cause CONTRACTION beta 2 will always cause RELAXATION

decreased Ach in what 2 diseases

alzhemiers huntingtons

In the sinoatrial (SA) node, phase 4 depolarization (pacemaker potential) is attributable to (A) an increase in K+ conductance (B) an increase in Na+ conductance (C) a decrease in Cl− conductance (D) a decrease in Ca2+ conductance (E) simultaneous increases in K+ and Cl− conductances

an increase in Na+ conductance Remember: the slope of phase 4 in the SA Node Action Potential is the one that determines heart rate. Sympathetic and Parasympathetic NS may affect this slope.

Sensory receptor potentials (A) are action potentials (B) always bring the membrane potential of a receptor cell toward threshold (C) always bring the membrane potential of a receptor cell away from threshold (D) are graded in size, depending on stimulus intensity (E) are all-or-none

are graded in size, depending on stimulus intensity

Thermoreceptors: A. are activated only by severe cold or severe heat B. are located on superficial layers of the skin C. are a subtype of nociceptors D. are on dendritic endings of Aδ fibers and C fibers E. All of the above

are on dendritic endings of Aδ fibers and C fibers Nociceptors and thermoreceptors are free nerve endings on unmyelinated or lightly myelinated fibers in hairy and glaborous skin and deep tissues. They are innervated by Aδ fibers and C fibers. Thermoreceptor is not a subtype of nociceptor and unlike nociceptor which is activated only in severe temperature condition, thermoreceptor threshold for hot and cold temperature is 30 and 24°Celsius respectively

Gap junctions A. are absent in cardiac muscle. B. are present but of little functional importance in cardiac muscle. C. are present and provide the pathway for rapid spread of excitation from one cardiac muscle fiber to another. D. are absent in smooth muscle. E. connect the sarcotubular system to individual skeletal muscle cells

are present and provide the pathway for rapid spread of excitation from one cardiac muscle fiber to another. AGAIN, GAP JUNCTIONS ARE RESPONSIBLE FOR SPREAD OF ACTION POTENTIAL / UNIFIED CONTRACTION / RAPID INTERCELLULAR TRANSPORT!!! HENCE, IT IS PRESENT IN CARDIAC MUSCLES AND IN UNITARY SMOOTH MUSCLES!

cardiac: 3 kinds of muscles

atrial and ventricle - (+) gap junctions, (+) syncytium pacemaker - (+) autorhythmiticity

Which of the following statements about nerve growth factor is not true? A. It is made up of three polypeptide subunits. B. It is responsible for the growth and maintenance of adrenergic neurons in the basal forebrain and the striatum. C. It is necessary for the growth and development of the sympathetic nervous system. D. It is picked up by nerves from the organs they innervate. E. It can express both p75 NTR and Trk A receptors. F. It is present in the brain

b Nerve Growth Factor is made up of three polypeptide subunits. It can be express both p75 NTR and Trk A receptors. It is necessary for the growth and development of the sympathetic nervous system; and cholinergic neurons in the basal forebrain and the striatum. It is picked up by nerves from the organs they innervate. It is present in the brain!

Primary function is planning and programming of movement

basal ganglia

what divides scala media from scala tympani?

basilar membrane this is where organ of corti sits (inside in scala media) this is aka frequency analyzer

BRAIN WAVES during REM

beta waves also present when awake; aroused; alert state; eyes open * B for Bibo!

aotic and ventricular pressure are highest in this part of the cardiac cycle

between rapid and reduced ventricular ejection

which is a TRUE action potential: Receptor Potential/Generator potential or "Slow waves" (to be discussed in the GI module)?

both are NOT a TRUE action potential! Merely brings you closer to threshold

Fast excitatory postsynaptic potentials (EPSPs): A. are a consequence of decreased Cl- conductance B. occurs in skeletal muscle C. can be produced by an increase in Na+ conductance D. can be produced by a decrease in Ca2+ conductance E. All of the above

can be produced by an increase in Na+ conductance The EPSP is produced by depolarization of the postsynaptic cell membrane immediately under the presynaptic ending. The excitatory transmitter opens Na+ or Ca2+ ion channels in the postsynaptic membrane, producing an inward current.

Hypoxemia produces hyperventilation by a direct effect on the (A) phrenic nerve (B) J receptors (C) lung stretch receptors (D) medullary chemoreceptors (E) carotid and aortic body chemoreceptors

carotid and aortic body chemoreceptors

where are baroreceptors found???

carotid sinus (CN9) - responds in INCREASE & DECREASE in BP aortic arch (CN10) - responds only in INCREASE in BP - result: will decrease HR to dec CO to dec BP --- ↑ BP → ↑ Stretch → ↑ Firing of CN IX to NTS → trigger parasympathetic response --- NOT carotid/aortic body! these are chemoreceptors!

Presbyopia presents with inability to read newspaper due to inability to contract what???

ciliary body Presbyopia: Convex Lenses used

What is a ganglion?

collection of neuron cell bodies (soma) outside CNS

in cardiomyocytes, this is increased by increasing phospholamban

concentration of Ca2+ within the SR → more Ca2+ accumulated by SR → more Ca2+ available for release to the sarcomere

highest concentration of light-sensitive cells, called cones, are found in??? greatest concentration of rods are found in the??

cones: MACULA rods: Parafoveal region

A 45-year-old woman who had never needed to wear glasses experienced difficulty reading a menu in a dimly-lit restaurant. She then recalled that as of late she needed to have the newspaper closer to her eyes in order to read it. A friend recommended she purchase reading glasses. Visual accommodation involves A. increased tension on the lens ligaments. B. a decrease in the curvature of the lens. C. relaxation of the sphincter muscle of the iris. D. contraction of the ciliary muscle. E. increased intraocular pressure.

contraction of the ciliary muscle. lens: SPHERE (near = sphere) suspensory ligament: relaxed ciliary muscle: contract Accommodation: PARASYMPATHETIC, uses MUSCARINIC receptors; CN3 stimulates ciliary muscle pupillary constriction occurs

The excessive muscle tone produced in decerebrate rigidity can be reversed by (A) stimulation of group Ia afferents (B) cutting the dorsal roots (C) transection of cerebellar connections to the lateral vestibular nucleus (D) stimulation of α-motoneurons (E) stimulation of γ-motoneurons

cutting the dorsal roots

How will Cribriform Plate Fracture affect olfaction???

damages CN I but not CN V (+) anosmia but (+) response to ammonia

A 30-year-old female patient's electrocardiogram (ECG) shows two P waves preceding each QRS complex. The interpretation of this pattern is: (A) decreased firing rate of the pacemaker in the sinoatrial (SA) node (B) decreased firing rate of the pacemaker in the atrioventricular (AV) node (C) increased firing rate of the pacemaker in the SA node (D) decreased conduction through the AV node (E) increased conduction through the His-Purkinje system

decreased conduction through the AV node

An electrocardiogram (ECG) on a person shows ventricular extrasystoles. The extrasystolic beat would produce (A) increased pulse pressure because contractility is increased (B) increased pulse pressure because heart rate is increased (C) decreased pulse pressure because ventricular filling time is increased (D) decreased pulse pressure because stroke volume is decreased (E) decreased pulse pressure because the PR interval is increased

decreased pulse pressure because stroke volume is decreased Remember the formula for Pulse Pressure is not just systolic pressure - diastolic pressure. The other formula is SV/arterial compliance. During extrasystolic beat (PVC), SV decreases due to premature contraction that lead to decreased ventricular filling time. Decreased SV means decreased pulse pressure (PP = SV/arterial compliance again).

A 55-year-old woman had an autonomic neuropathy that disrupted the sympathetic nerve supply to the pupillary dilator muscle of her right eye. While having her eyes examined, the ophthalmologist placed phenylephrine in her eyes. The right eye became much more dilated than the left eye. This suggests that A. the sympathetic nerve to the right eye had regenerated. B. the parasympathetic nerve supply to the right eye remained intact and compensated for the loss of the sympathetic nerve. C. phenylephrine blocked the pupillary constrictor muscle of the right eye. D. denervation supersensitivity had developed. E. the left eye also had nerve damage and so was not responding as expected.

denervation supersensitivity had developed. A good example of denervation supersensitivity is the response of the denervated iris. Hypersensitivity of the postsynaptic structure to the transmitter previously secreted by the axon endings is a general phenomenon, largely due to the synthesis or activation of more receptors.

DORSAL COLUMN-MEDIAL LEMNISCUS PATHWAY vs ANTERO-LATERAL SYSTEM (SPINOTHALAMIC TRACT) use what nerve fibers? which has temporal and spatial fidelity? decussation? which is for Vibration? 2 point discrimination? position sense? which is for pain and temp? Light Touch and Pressure Sensation?

dorsal column - Uses large myelinated fibers (Type II) - With temporal and spatial fidelity - Decussates near the medulla • Vibration • Sensations that signal movement against the skin • Position Sense and Fine Pressure • Two -Point Discrimination anterolateral spinothalamic - Uses smaller myelinated fibers (Type III, IV) - Decussates immediately • Pain • Temperature Sensation • Light Touch and Pressure Sensation • Tickle and Itch Sensation • Sexual Sensation SLAP SOMEONE IN THE FACE REALLY FAST His head will move in an anterolateral direction. There will be quick decussation of his head. He'll feel SLOW pain and temperature.

PO2 in left to right shunt

elevated on right side of the heart

Postrotatory nystagmus is caused by continued movement of A. aqueous humor over the ciliary body in the eye. B. cerebrospinal fluid over the parts of the brainstem that contain the vestibular nuclei. C. endolymph in the semicircular canals, with consequent bending of the cupula and stimulation of hair cells. D. endolymph toward the helicotrema. E. perilymph over hair cells that have their processes embedded in the tectorial membrane

endolymph in the semicircular canals, with consequent bending of the cupula and stimulation of hair cells. "The direction of the quick component during rotation is the same as that of the rotation, but the postrotatory nystagmus that occurs owing to displacement of the cupula when rotation is stopped is in the opposite direction."

endolymph is found in?? high in what electrolyte?? perilymph???

endolymph is seen in the scala media/cochlear duct - high in potassium perilymph is seen in the scala vestibuli and scala tympani - high in sodium

SARCOMERE entire length of myosin? pure myosin (no myosin heads)? purely actin?

entire length of myosin: A Band pure myosin (no myosin heads): H Band purely actin: I band

Olfactory memories are found in what part of the brain??

entorhinal cortex

If an area of the lung is not ventilated because of bronchial obstruction, the pulmonary capillary blood serving that area will have a PO2 that is (A) equal to atmospheric PO2 (B) equal to mixed venous PO2 (C) equal to normal systemic arterial PO2 (D) higher than inspired PO2 (E) lower than mixed venous PO2

equal to mixed venous PO2

ECG can basically be divided into 2 - everything that happens to atrium aka ____ and everything that happens to ventricles aka ____

everything that happens to atrium aka PR interval everything that happens to ventricles aka QT interval

Sound Frequency measured in? Directly correlated with? what range can humans hear?

frequenZy Hertz PITCH Human ear: 20-20,000 Hz The loudness of a sound is directly correlated with the amplitude of a sound wave, and pitch is directly correlated with the frequency of the sound wave.

WHAT LAYER OF RETINA: Axons combine to form optic nerve

ganglion cell layer

The low-resistance pathways between myocardial cells that allow for the spread of action potentials are the (A) gap junctions (B) T tubules (C) sarcoplasmic reticulum (SR) (D) intercalated disks (E) mitochondria

gap junctions

why does Tetanus / Tetanic Spasm happen

happens when all Ca2+ from the SR has been released; no further increase in muscle strength

A 28-year-old man with severe myopia made an appointment to his ophthalmologist when he began to notice flashing lights and floaters in his visual field. He was diagnosed with a retinal detachment. The retina A. is epithelial tissue that contains photoreceptors. B. lines the anterior one-third of the choroid. C. has an inner nuclear layer that contains bipolar cells, horizontal cells, and see amacrine cells. D. contains ganglion cells whose axons form the oculomotor nerve. E. contains an optic disk where visual acuity is greatest

has an inner nuclear layer that contains bipolar cells, horizontal cells, and see amacrine cells. -- "The retina extends anteriorly almost to the ciliary body. It is organized in 10 layers and contains the rods and cones, which are the visual receptors, plus four types of neurons: bipolar cells, ganglion cells, horizontal cells, and amacrine cells." "It contains ganglion cells whose axons form the optic nerve. The area of the optic disk contains no visual receptors over the disk, and consequently this spot is blind (the blind spot)."

Cell membranes: A. contain relatively few protein molecules. B. contain many carbohydrate molecules. C. are freely permeable to electrolytes but not to proteins. D. have variable protein and lipid contents depending on their location in the cell. E. have a stable composition throughout the life of the cell.

have variable protein and lipid contents depending on their location in the cell. "Cellular Membranes have variable protein and lipid contents depending on their location in the cell and is semipermeable, allowing some substances to pass through it and excluding others. There were many proteins embedded in the membrane with abundant phospholipids." Cell membrane have variable composition throughout life of the cell.

Turbulent Blood Flow: irregular, disorderly blood flow associated with _____ and ______

high Reynold's Number (>2000) & bruits (audible vibrations)

When a person is standing, blood flow in the lungs is (A) equal at the apex and the base (B) highest at the apex owing to the effects of gravity on arterial pressure (C) highest at the base because that is where the difference between arterial and venous pressure is greatest (D) lowest at the base because that is where alveolar pressure is greater than arterial pressure

highest at the base because that is where the difference between arterial and venous pressure is greatest

An inhibitory postsynaptic potential: (A) depolarizes the postsynaptic membrane by opening Na+ channels (B) depolarizes the postsynaptic membrane by opening K+ channels (C) hyperpolarizes the postsynaptic membrane by opening Ca2+ channels (D) hyperpolarizes the postsynaptic membrane by opening Cl- channels

hyperpolarizes the postsynaptic membrane by opening Cl- channels Remember - you can inhibit an AP several ways - cause Cl- influx, or cause K+ efflux or inhibit Na+ influx/Ca2+ influx. All of these will make the cell more negative/less positive.

what will happen to RBC when placed in hypertonic solution? hypotonic solution? isotonic?

hypertonic - rbc will SHRINK hypotonic - rbc will SWELL isotonic - no change

STARLING EQUATION normal net filtration??????

if Positive, promotes filtration (fluid moves out of the capillary) If Negative, promotes absorption (fluid moves into the capillary) ** KF = KaFillary Fermeability normal net filtration = 2mL/min

A lesion of the chorda tympani nerve would most likely result in (A) impaired olfactory function (B) impaired vestibular function (C) impaired auditory function (D) impaired taste function (E) nerve deafness

impaired taste function Anterior 2/3 of tongue: CN 7 (chorda tympani Facial Nerve) Posterior 1/3 of tongue: CN 9 (Glossopharyngeal Nerve) Back of throat and epiglottis: CN 10 (Vagus Nerve)

An electrocardiogram (ECG) on a person shows ventricular extrasystoles. After an extrasystole, the next "normal" ventricular contraction produces (A) increased pulse pressure because the contractility of the ventricle is increased (B) increased pulse pressure because total peripheral resistance (TPR) is decreased (C) increased pulse pressure because compliance of the veins is decreased (D) decreased pulse pressure because the contractility of the ventricle is increased (E) decreased pulse pressure because TPR is decreased

increased pulse pressure because the contractility of the ventricle is increased After PVC, there is increased ventricular filling time, increasing SV. This will increase Pulse pressure (PP = SV/arterial compliance once again)

Fast inhibitory postsynaptic potentials (IPSPs): A. are a consequence of decreased Cl- conductance B. occurs in skeletal muscle C. can be produced by an increase in Na+ conductance D. can be produced by a decrease in Ca2+ conductance E. interact with other fast and slow potentials to move the membrane potential of the postsynaptic neuron toward or away from the firing level

interact with other fast and slow potentials to move the membrane potential of the postsynaptic neuron toward or away from the firing level Fast inhibitory postsynaptic potentials (IPSPs) can be produced by a localized increase in Cl- transport and by opening of K+ channels, with movement of K+ out of the postsynaptic cell, or by closure of Na+ or Ca2+ channels. It occurs in the motor neuron presynaptic ending and interact with other fast and slow potentials to move the membrane potential of the postsynaptic neuron toward or away from the firing level.

detects changes in Muscle Length

intrafusal! muscle spindle (innervation: gamma-motorneurons) versus Extrafusal: for voluntary muscle contraction (innervation: alpha-motorneurons) AEGIS: Alpha-Extrafusal, Gamma-Intrafusal

The Golgi complex A. is an organelle that participates in the breakdown of proteins and lipids. B. is an organelle that participates in posttranslational processing of proteins. C. is an organelle that participates in energy production. D. is an organelle that participates in transcription and translation. E. is a subcellular compartment that stores proteins for trafficking to the nucleus.

is an organelle that participates in posttranslational processing of proteins • Golgi complex is an organelle that participates in posttranslational processing of proteins. • Lysosome is an organelle that participates in the breakdown of proteins and lipids. • Mitochondrion is an organelle that participates in energy production. • Nucleus is an organelle that participates in transcription and translation. • Ribosomal Endoplasmic Reticulum is a subcellular compartment that stores proteins for trafficking to the nucleus.

The auditory pathway A. and vestibular pathway contain a synapse in the cerebellum. B. and vestibular pathway project to the same regions of the cerebral cortex. C. is composed of afferent fibers of the eighth cranial nerve, the dorsal and ventral cochlear nuclei, the superior colliculi, the lateral geniculate body, and the auditory cortex. D. is composed of afferent fibers of the eighth cranial nerve, the dorsal and ventral cochlear nuclei, the inferior colliculi, the medial geniculate body, and the auditory cortex. E. is not subject to plasticity like the visual pathways.

is composed of afferent fibers of the eighth cranial nerve, the dorsal and ventral cochlear nuclei, the inferior colliculi, the medial geniculate body, and the auditory cortex. -- "The activity within the auditory pathway passes from the eighth cranial nerve afferent fibers to the dorsal and ventral cochlear nuclei to the inferior colliculi to the thalamic medial geniculate body and then to the auditory cortex."

A 62-year-old white woman experienced a rapid onset of blurry vision along with loss of central vision. A comprehensive eye exam showed that she had wet age-related macular degeneration. The fovea of the eye A. has the lowest light threshold. B. is the region of highest visual acuity. C. contains only red and green cones. D. contains only rods. E. is situated over the head of the optic nerve.

is the region of highest visual acuity. "Near the posterior pole of the eye is a yellowish pigmented spot, the MACULA LUTEA This marks the location of the fovea centralis, a thinned- out, rod-free portion of the retina that is present in humans and other primates. In it, the cones are densely packed, and each synapse to a single bipolar cell, which, in turn, synapses on a single ganglion cell, providing a direct pathway to the brain. Consequently, the fovea is the point where visual acuity is greatest."

ventricular volume is highest at this phase of cardiac cycle

isovolumic contraction

phase of cardiac cycle where all 4 valves are closed

isovolumic contraction isovolumic relaxation

A ventrolateral cordotomy is performed that produces relief of pain in the right leg. It is effective because it interrupts the A. left dorsal column. B. left ventrolateral spinothalamic tract. C. right ventrolateral spinothalamic tract. D. right medial lemniscal pathway. E. a direct projection to the primary somatosensory cortex.

left ventrolateral spinothalamic tract The axons from the neurons responsible for nociception crosses the midline and ascend in the ventrolateral quadrant of the spinal cord, where they form the ventrolateral spinothalamic tract; therefore, ventrolateral cordotomy will result in loss of pain sensation in the contralateral side.

WHAT HAPPENS TO THE FOLLOWING IN ACCOMMODATION (looking at near objects) lens? suspensory ligament? ciliary muscle?

lens: SPHERE (near = sphere) suspensory ligament: relaxed ciliary muscle: contract Accommodation: PARASYMPATHETIC, uses MUSCARINIC receptors; CN3 stimulates ciliary muscle pupillary constriction occurs

A 42-year-old man with myasthenia gravis notes increased muscle strength when he is treated with an acetylcholinesterase (AChE) inhibitor. The basis for his improvement is increased (A) amount of acetylcholine (ACh) released from motor nerves (B) levels of ACh at the muscle end plates (C) number of ACh receptors on the muscle end plates (D) amount of norepinephrine released from motor nerves (E) synthesis of norepinephrine in motor nerves

levels of ACh at the muscle end plates Neostigmine is part of the treatment for MG. By inhibiting AChase, ACh levels will increase, decreasing muscle weakness.

During exercise, total peripheral resistance (TPR) decreases because of the effect of (A) the sympathetic nervous system on splanchnic arterioles (B) the parasympathetic nervous system on skeletal muscle arterioles (C) local metabolites on skeletal muscle arterioles (D) local metabolites on cerebral arterioles (E) histamine on skeletal muscle arterioles

local metabolites on skeletal muscle arterioles

A young boy was diagnosed with congenital anosmia, a rare disorder in which an individual is born without the ability to smell. Odorant receptors are A. located in the olfactory bulb. B. located on dendrites of mitral and tufted cells. C. located on neurons that project directly to the olfactory cortex. D. located on neurons in the olfactory epithelium that project to mitral cells and from there directly to the olfactory cortex. E. located on sustentacular cells that project to the olfactory bulb.

located on neurons in the olfactory epithelium that project to mitral cells and from there directly to the olfactory cortex. "Olfactory sensory neuron expresses only one of the 1000 different odorant receptors. Each neuron projects to one or two glomeruli which provides a distinct two-dimensional map in the olfactory bulb that is unique to the odorant. The mitral cells with their glomeruli project to different parts of the olfactory cortex."

where do you find norepi?

locus cereleus iLOCUS NortE

A person with a ventilation/perfusion (V/Q) defect has hypoxemia and is treated with supplemental O2. The supplemental O2 will be most helpful if the person's predominant V/Q defect is (A) dead space (B) shunt (C) high V/Q (D) low V/Q (E) V/Q=0 (F) V/Q=×

low V/Q Low V/Q here means mababa yung ventilation, or mataas yung perfusion or both. So, if you give O2, naimprove mo yung oxygenation ng blood (at dahil pwedeng mataas yung perfusion, mas appreciated yung supplemental O2). Hindi pwedeng E yung sagot dito - no ventilation means no effect at all with supplemental O2.

Most important role of Gamma Motoneurons

maintain 1a afferent activity during muscle contraction

memory storage mainly in?? encodes new memory?? helps recall previously formed memories?? what will lead to anterograde amnesia when destroyed?? to retrograde amnesia??

memory is stored throughout the brain but mainly in the temporal lobe (Temporal = sTorage) Hippocampus helps ENCODE or form new memory Thalamus meanwhile helps you RECALL previously-formed memories. Destroying your hippocampus would cause ANTEROGRADE amnesia (cannot form new memories). Destroying your thalamus would cause RETROGRADE amnesia (cannot recall old memories).

reflex example of: monosynaptic disynaptic polysynaptic

mono: Stretch Reflex (Knee- Jerk) - Contraction of the muscle di: Golgi-Tendon Reflex (Clasp Knife) also called INVERSE STRETCH REFLEX - Relaxation of the muscle poly: Flexor-Withdrawal Reflex (after touching ahot stove) - Ipsilateral flexion; contralateral extension

motor unit size principle

motor units are recruited in order from small to large Small MU active first before large MU to make sure that you have muscle contraction that starts as weak, and gradually becomes strong (you don't want the opposite to happen - if the opposite occurs, it will mess up your movements). MOTOR UNIT = single motoneuron + all the muscle fibers that it innervates

A 35-year-old woman sees her physician to report muscle weakness in the extraocular eye muscles and muscles of the extremities. She states that she feels fine when she gets up in the morning, but the weakness begins soon after she becomes active. The weakness is improved by rest. Sensation appears normal. The physician treats her with an anticholinesterase inhibitor, and she notes immediate return of muscle strength. Her physician diagnoses her with A. Lambert-Eaton syndrome. B. myasthenia gravis. C. multiple sclerosis. D. Parkinson disease. E. muscular dystrophy.

myasthenia gravis. Myasthenia gravis is caused by the formation of circulating antibodies to the muscle type of nicotinic acetylcholine receptors this leads to the major clinical feature of the disease-muscle fatigue with sustained or repeated activity. In contrast, Lambert-Eaton Syndrome which is a condition wherein muscle weakness is caused by an autoimmune attack against one of the Ca2+ channels in the nerve at the neuromuscular junction, improves with repetitive stimulation. Among the choices, only myasthenia will have clinical improvement after administration of an anticholinesterase inhibitor.

The velocity of conduction of action potentials along a nerve will be increased by __________. (A) stimulating the Na+-K+ pump (B) inhibiting the Na+-K+ pump (C) decreasing the diameter of the nerve (D) myelinating the nerve (E) lengthening the nerve fiber

myelinating the nerve

Conduction velocity of nerve action potential is most dependent on?

nerve diameter

Characteristic of CO poisoning to PaO2 and Arterial O2 saturation

normal PaO2, lower than normal Arterial O2 saturation which is why it results to Anemic hypoxia

what pulmo capillary pressure will lead to edema??

normal pulmonary capillary pressure: 7mmHg at 28mmHg, there will be edema! 7-28 mmHg (buffer pressure)

A ballet dancer spins to the left. During the spin, her eyes snap quickly to the left. This fast eye movement is (A) nystagmus (B) postrotatory nystagmus (C) ataxia (D) aphasia

nystagmus Nystagmus - same direction as head rotation. Postrotatory nystagmus - opposite direction - Due to continued movement of endolymph in the semicircular canals, with consequent bending of the cupula and stimulation of hair cells

3 Sites with little/no parasympathetic innervation:

o Cardiac Ventricles o Pregnant Uterus o Blood Vessels

BBB consists of what (3)

o Endothelial cells of cerebral capillaries (and the tight junctions between them) o Astrocyte foot processes o Choroid plexus epithelium

Place Theory of Hearing

o Inner hair Cells near BASE (oval and round windows): respond to high-frequency sounds o Inner hair Cells near Apex (helicotrema): respond to low-frequency sounds

Sympathetic distribution, but final NT is Ach and final receptor is Muscarinic (3)

o Sweat glands (some, not all) o Piloerector muscles (controversial, utilizes 𝛼1 according to some sources) o 𝜷3 Receptors: seen in Brown Adipose Tissue of babies

what happens to functional residual capacity in OBSTRUCTIVE lung disease? in restrictive???

obstructive: INCREASED (trapped air) restrictive: DECREASED (air cant go in)

reflection coefficient - what does it mean when substance has Rc of: one? (example?) between zero to one? zero? (example?)

one - NO solute penetration (EFFECTIVE osmole meaning it attracts water!) - example: albumin! between zero to one - some solute penetration - most substances zero: COMPLETE solute penetration (INEFFECTIVE osmole) - example: urea (think: if ZERO ka, COMPLETEly INEFFECTIVE ka, like UREA, iniihi mo lang kasi walang kwenta)

rods and cones are absent in???

optic disk BLIND SPOT!

low to high concentration - what transport?

osmosis primary active transport secondary active transport

EFFECTIVE OSMOTIC PRESSURE

osmotic pressure x reflection coefficient determines whether solution is hypo, hyper, isotonic ** Rule: water undergoes osmosis from HYPOtonic solution to HYPERtonic solution

If a patient is unable to hear high-frequency sound, damage is closest to

oval window

The tendency for blood flow to be turbulent is increased by (A) increased viscosity (B) increased hematocrit (C) partial occlusion of a blood vessel (D) decreased velocity of blood flow

partial occlusion of a blood vessel

cardiac action potential main electrolyte per phase cardiac pacemaker action potential main electrolyte per phase

phase 0 - Na phase 1 - K phase 2 - Ca phase 3 - K phase 4 (none, resting membrane potential) phase 4 - Na (funny Na channels) phase 0 - Ca phase 3 - K

Cardiac Action Potential phase 2

phase 2 - pla2 (plateau) - Ca2+ in = K out this prolongs the action potential

pneumotaxic vs apneustic centers found where? what are the effects?

pneumotaxic @ upper pons apneustic @ lower pons Pneumotaxic Center - think "pneumonia" - pampabilis ng paghinga - it increases respiratory rate. Apneustic center - think "apnea" - pampabagal ng paghinga - it decreases respiratory rate.

what phase in cardiac cycle will the ff happen: preceded by P wave preceded by QRS wave preceded by T wave

preceded by P wave: ATRIAL CONTRACTION preceded by QRS wave: ISOVOLUMIC CONTRACTION preceded by T wave: ISOVOLUMIC RELAXATION

what kind of nerve fiber do your preganglionic nerve fibers have? postganglionic?

preganglionic - Type B postganglionic - Type C

Law of Laplace

pressure is directly proportional to surface tension and inversely proportional to radius of alveolus

Sound Pressure measured in? Directly correlated with? ___ dB: conversational Speech ___ dB: limit to prevent Occupational Hearing Loss ___ dB: causes pain, triggers attenuation reflex (stapedius and tensor tympani contract reflexively)

pressure!! Decibels (dB) AMPLITUDE (loudness/clarity) ** pressure dBa?! amppp 60dB: conversational Speech 85 dB: limit to prevent Occupational Hearing Loss >120 dB: causes pain, triggers attenuation reflex (stapedius and tensor tympani contract reflexively - malleus pulled inward; stapes pulled outward) The loudness of a sound is directly correlated with the amplitude of a sound wave, and pitch is directly correlated with the frequency of the sound wave.

The contractile response in skeletal muscle A. starts after the action potential is over. B. does not last as long as the action potential. C. produces more tension when the muscle contracts isometrically than when the muscle contracts isotonically. D. produces more work when the muscle contracts isometrically than when the muscle contracts isotonically. E. decreases in magnitude with repeated stimulation.

produces more tension when the muscle contracts isometrically than when the muscle contracts isotonically. - Muscle fiber does not have a refractory period, repeated stimulation before relaxation produces additional activation of the contractile elements, or summation of contractions. Note that because work is the product of force times distance, isotonic contractions do work, whereas isometric contractions do not. When the muscle fiber contracts isometrically, the tension developed is proportional to the number of cross-bridges between the actin and the myosin molecules Contraction starts before the action potential is over and last longer as the action potential.

A 40-year-old man loses his right hand in a farm accident. Four years later, he has episodes of severe pain in the missing hand (phantom limb pain). A detailed PET scan study of his cerebral cortex might be expected to show A. expansion of the right-hand area in his right primary somatosensory cortex. B. expansion of the right-hand area in his left primary somatosensory cortex. C. a metabolically inactive spot where his hand area in his left primary somatosensory cortex would normally be. D. projection of fibers from neighboring sensory areas into the right-hand area of his right primary somatosensory cortex. E. projection of fibers from neighboring sensory areas into the right-hand area of his left primary somatosensory cortex

projection of fibers from neighboring sensory areas into the right-hand area of his left primary somatosensory cortex

F-actin is a component of the cellular cytoskeleton that: A. provides a structural component for cell movement. B. is defined as the "functional" form of actin in the cell. C. refers to the actin subunits that provide the molecular building blocks of the extended actin molecules found in the cell. D. provides the molecular architecture for cell to cell communication.

provides a structural component for cell movement -- Filamentous (F) actin provides a structural component for cell movement. Globular (G) actin refers to the actin subunits that provide the molecular building blocks of the extended actin molecules found in the cell. Microtubules provides the molecular architecture for cell to cell communication. Intermediate filaments form a flexible scaffolding for the cell and help it resist external pressure.

Only arteries that contain deoxygenated blood? (2)

pulmonary artery and umbilical artery

phase of cardiac cycle where ventricular pressure > aortic pressure

rapid ventricular ejection

Abnormal color vision is 20 times more common in males than females because most cases are caused by an abnormal A. dominant gene on the Y chromosome. B. recessive gene on the Y chromosome. C. dominant gene on the X chromosome. D. recessive gene on the X chromosome. E. recessive gene on chromosome 22.

recessive gene on the X chromosome. "Color blindness are inherited as recessive and X-linked characteristics. Color blindness is present in males if the X chromosome has the abnormal gene. Females show a defect only when both X chromosomes contain the abnormal gene."

Endocytosis A. includes phagocytosis and pinocytosis, but not clathrin mediated or caveolae-dependent uptake of extracellular contents. B. refers to the merging of an intracellular vesicle with the plasma membrane to deliver intracellular contents to the extracellular milieu. C. refers to the invagination of the plasma membrane to uptake extracellular contents into the cell. D. refers to vesicular trafficking between Golgi stacks.

refers to the invagination of the plasma membrane to uptake extracellular contents into the cell. • "Endocytosis refers to the invagination of the plasma membrane to uptake extracellular contents into the cell. It includes phagocytosis, pinocytosis, clathrin mediated or caveolae dependent uptake of extracellular contents." • "Exocytosis refers to vesicular trafficking between Golgi stacks. The process refers to the merging of an intracellular vesicle with the plasma membrane to deliver intracellular contents to the extracellular milieu."

In the utricle, tip links in hair cells are involved in A. formation of perilymph. B. depolarization of the stria vascularis. C. movements of the basement membrane. D. perception of sound. E. regulation of distortion-activated ion channels.

regulation of distortion-activated ion channels. When a stereocilium is pushed toward a taller stereocilium, the tip line is stretched and opens an ion channel in its taller neighbor. The channel next is presumably moved down the taller stereocilium by a molecular motor, so the tension on the tip link is released. When the hairs return to the resting position, the motor moves back up the stereocilium.

maintains oxygenation in between breaths

residual volume

Main difference in contraction of smooth muscles vs. skeletal muscles

role of Ca2+ in initiating contraction

Functional and structural unit of a muscle of skeletal and cardiac muscles

sarcomere from one Z line to another

hormone released as a result of blood vessel damage; causes arteriolar vasoconstriction; implicated in migraine

serotonin

Which of the following synaptic transmitters is NOT a peptide, polypeptide, or protein? A. substance P B. met-enkephalin C. Beta-endorphin D. serotonin E. dynorphin

serotonin Small molecule transmitters include monoamines (e.g., acetylcholine, serotonin, histamine), catecholamines (dopamine, norepinephrine, and epinephrine), and amino acids (e.g., glutamate, GABA, glycine). Large-molecule transmitters include a large number of peptides called neuropeptides including substance P, enkephalin, vasopressin.

Gap junctions are intercellular connections that A. primarily serve to keep cells separated and allow for transport across a tissue barrier. B. serve as a regulated cytoplasmic bridge for sharing of small molecules between cells. C. serve as a barrier to prevent protein movement within the cellular membrane. D. are cellular components for constitutive exocytosis that occurs between adjacent cells.

serve as a regulated cytoplasmic bridge for sharing of small molecules between cells Tight junctions provide intercellular connections that link cells into a regulated tissue barrier, it also provides BARRIER TO MOVEMENT OF PROTEINS in the cell membrane and thus, are important to cellular polarization. DIVIDES APICAL FROM BASOLATERAL! Gap junctions provide contacts between cells that allow for direct passage of small molecules between two cells. FOR RAPID INTERCELLULAR TRANSPORT - ACTION POTENTIAL PROPAGATION! Desmosomes and adherens junctions are specialized structures that hold cells together. LIKE STAPLER! Hemidesmosomes and focal adhesions attach CELL TO BASAL LAMINA

A 27-year-old man was brought to the emergency department with symptoms of opioid intoxication. He was given an intravenous dose of naloxone. Endogenous opioids A. bind to both ionotropic receptors and GPCR. B. include morphine, endorphins, and dynorphins. C. show the following order of affinity for δ receptors: dynorphins > > endorphins. D. show the following order of affinity for μ receptors: dynorphins > endorphins. E. show the following order of affinity for κ receptors: endorphins > > enkephalins.

show the following order of affinity for δ receptors: dynorphins > > endorphins o Mu: Endorphins > Enkephalins > Dynorphin o Kappa: Enk > End and Dy o Delta: Dyn > End and Enk

high to low concentration - what transport?

simple DIFFUSION facilitated DIFFUSION

what determines heart rate??

slope of phase 4 of the pacemaker cells - If (funny) Na channels if increased Na channel influx, steeper slope, will reach the threshold faster, increase HR

"Calcium-Induced Calcium-Release" system seen in what muscles

smooth and cardiac muscles

Spinal cord main inhibitory neurotransmitter? Brain main inhibitory neurotransmitter?

spinal cord: GLYCINE brain: GABA Y in glycine looks like spinal cord B in gaBa is for Brain!

The action potential of skeletal muscle: has a prolonged plateau phase. spreads inward to all parts of the muscle via the T tubules. causes the immediate uptake of Ca2+ into the lateral sacs of the sarcoplasmic reticulum. is longer than the action potential of cardiac muscle. is not essential for contraction.

spreads inward to all parts of the muscle via the T tubules Action potential is essential for contraction which spreads inward to all parts of the muscle via the T- tubules; it triggers the release of Ca2+ from the terminal cisterns, the lateral sacs of the sarcoplasmic reticulum next to the T system; it is shorter than the action potential of cardiac muscle (which has prolonged plateau phase).

The correct sequence of events involved in phototransduction in rods and cones in response to light is: A. activation of transducin, decreased release of glutamate, structural changes in rhodopsin, closure of Na+ channels, and decrease in intracellular cGMP. B. decreased release of glutamate, activation of transducin, closure of Na+ channels, decrease in intracellular cGMP, and structural changes in rhodopsin. C. structural changes in rhodopsin, decrease in intracellular cGMP, decreased release of glutamate, closure of Na+ channels, and activation of transducin. D. structural changes in rhodopsin, activation of transducin, decrease in intracellular cGMP, closure of Na+ channels, and decreased release of glutamate. E. activation of transducin, structural changes in rhodopsin, closure of Na+ channels, decrease in intracellular cGMP, and decreased release of glutamate.

structural changes in rhodopsin, activation of transducin, decrease in intracellular cGMP, closure of Na+ channels, and decreased release of glutamate.

A 47-year-old woman was admitted to the hospital after experiencing nausea and vomiting for about 2 days followed by severe muscle weakness and neurologic symptoms, including ptosis and dysphagia. She indicated she had eaten at a restaurant the evening before the symptoms began. Laboratory tests were positive for Clostridium botulinum. Neurotoxins A. block the reuptake of neurotransmitters into presynaptic terminals. B. such as tetanus toxin bind reversibly to the presynaptic membrane at the neuromuscular junction. C. reach the cell body of the motor neuron by diffusion into the spinal cord. D. exert all of their adverse effects by acting centrally rather than peripherally. E. such as botulinum toxin prevent the release of acetylcholine from motor neurons due to cleavage of either synaptosome-associated proteins or vesicle-associated membrane proteins.

such as botulinum toxin prevent the release of acetylcholine from motor neurons due to cleavage of either synaptosome-associated proteins or vesicle-associated membrane proteins. - Clinically, tetanus toxin causes spastic paralysis by blocking presynaptic transmitter release in the CNS, and botulism causes flaccid paralysis by blocking the release of acetylcholine at the neuromuscular junction.

Complete transection of the spinal cord at the level of T1 would most likely result in (A) temporary loss of stretch reflexes below the lesion (B) temporary loss of conscious proprioception below the lesion (C) permanent loss of voluntary control of movement above the lesion (D) permanent loss of consciousness above the lesion

temporary loss of stretch reflexes below the lesion

The pH of venous blood is only slightly more acidic than the pH of arterial blood because (A) CO2 is a weak base (B) there is no carbonic anhydrase in venous blood (C) the H+ generated from CO2 and H2O is buffered by HCO3- in venous blood (D) the H+ generated from CO2 and H2O is buffered by deoxyhemoglobin in venous blood (E) oxyhemoglobin is a better buffer for H+ than is deoxyhemoglobi

the H+ generated from CO2 and H2O is buffered by HCO3- in venous blood

Positive Staircase / Bowditch Staircase / Treppe

the faster the heart beats, the stronger the heart beats

During the upstroke of the nerve action potential (A) there is net outward current and the cell interior becomes more negative (B) there is net outward current and the cell interior becomes less negative (C) there is net inward current and the cell interior becomes more negative (D) there is net inward current and the cell interior becomes less negative

there is net inward current and the cell interior becomes less negative

The greatest pressure decrease in the circulation occurs across the arterioles because (A) they have the greatest surface area (B) they have the greatest cross-sectional area (C) the velocity of blood flow through them is the highest (D) the velocity of blood flow through them is the lowest (E) they have the greatest resistance

they have the greatest resistance

Minute Ventilation

tidal volume x respiratory rate

Solutions A and B are separated by a membrane that is permeable to urea. Solution A is 10 mM urea, and solution B is 5 mM urea. If the concentration of urea in solution A is doubled, the flux of urea across the membrane will (A) double (B) triple (C) be unchanged (D) decrease to one-half (E) decrease to one-third

triple For the guide question above: J1 = PA (C1-C2) =PA (10-5) = 5 J2 = PA (C1-C2) = PA (20-5) = 15 J2 is 3x more than J1. Kaya "triple" yung sagot.

2 types of extrafusal muscles (type 1 and type 2)

type 1 - slow twitch/red muscle for endurance! e.g. back/postural muscles oxidative! red coz more myoglobin "ONE SLOW RED OX with perfect posture" type 2 - fast twitch/white muscle

thermoreceptors and nociceptors use what sensory receptors (2)

type A delta type C warmth threshold: 30°C cold threshold: 24°C, cold receptors inactive again at <10°C

DAMAGE TO Inferior parietal lobule (a region in the posterior part of the parietal lobe that is close to the occipital lobe) will lead to?

unilateral attention and neglect

The correct temporal sequence for events at the neuromuscular junction is (A) action potential in the motor nerve; depolarization of the muscle end plate; uptake of Ca2+ into the presynaptic nerve terminal (B) uptake of Ca2+ into the presynaptic terminal; release of acetylcholine (ACh); depolarization of the muscle end plate (C) release of ACh; action potential in the motor nerve; action potential in the muscle (D) uptake of Ca2+ into the motor end plate; action potential in the motor end plate; action potential in the muscle (E) release of ACh; action potential in the muscle end plate; action potential in the muscle

uptake of Ca2+ into the presynaptic terminal; release of acetylcholine (ACh); depolarization of the muscle end plate

Van't Hoff's Law

used to calculate OSMOTIC PRESSURE

Thromboxane A2 2 effects

vasoconstriction and platelet aggregation Its natural inhibitor is PROSTACYCLIN.

skeletal vs cardiac vs smooth muscle which has sarcomeres? which has troponin? which has plateau on action potential? which has gap junction? which has greatest sarcoplasmic reticulum?

which has sarcomeres? SKELETAL & CARDIAC which has troponin? SKELETAL & CARDIAC which has plateau on action potential? CARDIAC (muscles only like atria, ventricles, purkinje fibers! not pacemaker) which has gap junction? CARDIAC & UNITARY SMOOTH MUSCLES which has greatest sarcoplasmic reticulum? SKELETAL

A healthy male medical student volunteered to undergo evaluation of the function of his vestibular system for a class demonstration. The direction of his nystagmus is expected to be vertical when he is rotated A. after warm water is put in one of his ears. B. with his head tipped backward. C. after cold water is put in both of his ears. D. with his head tipped sideways. E. with his head tipped forward.

with his head tipped sideways. Nystagmus is frequently horizontal (i.e. the eyes move in the horizontal plane), but it can also be vertical (when the head is tipped sidewise during rotation) or rotatory (when the head is tipped forward).

water osmolarity?

zero! OSMOLARITY: Concentration of osmotically active particles in a solution Remember: water utilizes OSMOSIS and not simple diffusion. Water will move according to CONCENTRATION GRADIENT (concentration differences between two compartments) and not according to volume differences between two compartments. Water will move from LOW CONCENTRATION ("dilute" compartment) to HIGH-CONCENTRATION ("concentrated" compartment). Do not confuse water movement (osmosis) with solute movement using simple diffusion.

Receptive field types

§ Type 1 Receptive Field: smaller with well-defined borders § Type 2 Receptive Field: wider but with poorly-defined borders - Don't ignore these. A receptive field is a region controlled by a single sensory neuron. Pag type 1, small area but with well-defined borders. Pag type 2, large areas, pero poorly defined ang borders. Which is therefore best for 2-point discrimination (ability to detect two separate touch stimuli as two discrete points)? Type 1 is the answer - kasi smaller areas with well-defined borders ang kontrolado ng bawat sensory neuron na may Type 1 receptive field.

Which receptor mediates constriction of arteriolar smooth muscle? (A) α1 Receptors (B) β1 Receptors (C) β2 Receptors (D) Muscarinic receptors

α1 Receptors Remember our discussion in the Neuro module. Alpha-1 causes smooth muscle contraction. Beta-2 causes smooth muscle relaxation. Alpha-1 causes vasoconstriction. Beta-2 causes vasodilation.

what part of hypothalamus leads to heat loss? heat gain???

• Anterior Hypothalamus: for heat loss • Posterior Hypothalamus: for heat gain think of yourself fanning the anterior part of the body. ANTERIOR hypothalamus: heat loss. Think of yourself placing on a jacket to keep yourself warm. You first place that jacket on the posterior part of the body. POSTERIOR hypothalamus: heat gain/preservation.

parts of neuron: • ______: where NT receptors are found • ______: where organelles, nucleus is seen • ______: where AP starts • ______: transmitting portion • ______: unmyelinated portion of the axon • ______: branches of the axon synaptic transmission occurs from where to where?

• Dendrites: where NT receptors are found • Cell Body (Soma): where organelles, nucleus is seen • Initial Segment (axon hillock): where AP starts • Axon: transmitting portion • Nodes of Ranvier: unmyelinated portion of the axon - responsible for SALTATORY CONDUCTION • Neural Fibril: branches of the axon Orthodromic (Synapse to Axon) rather than Antidromic (Axon to Synapse)

Which of the following receptors is correctly identified as an ionotropic or a G-protein-coupled receptor (GPCR)? A. Neurokinin receptor: ionotropic B. Nicotinic receptor: GPCR C. GABA A receptor: ionotropic D. NMDA receptor: GPCR E. Glycine: GPCR

• GABAA receptor: ionotropic (chloride) • GABAB receptor: metabotropic/GPCR • Neurokinin receptor: metabotropic/GPCR • Nicotinic receptor: ionotropic (sodium/potassium) • NMDA receptor: ionotropic (calcium) • Glycine: ionotropic (chloride)

GLUT transporters found where??

• GLUT 1: BBB • GLUT 2: Liver, Pancreas, Basement Membrane of SI • GLUT 3: Neurons • GLUT 4: Muscles, Adipose • GLUT 5: for fructose transport from SI lumen to SI cell

spatial vs temporal summation? posttetanic stimulation?

• Spatial Summation: 2 or more excitatory inputs at the same time(A+B+C) • Temporal summation: 2 or more excitatory inputs at rapid succession (A...A...A...) • Facilitation / Augmentation / Postetanic Stimulation: brings cell closer to threshold

A 47-year-old woman experienced migraine headaches that were not relieved by her current pain medications. Her doctor prescribed one of the newer analgesic agents that exert their effects by targeting synaptic transmission in nociception and peripheral sensory transduction. Which of the following drugs is correctly paired with the type of receptor it acts on to exert its antinociceptive effects? A. Topiramate and Na+ channel B. Ziconotide and NMDA receptors C. Naloxone and opioid receptors D. Lidocaine and TRPVI channels E. Gabapentin and Nav1.8

• Topiramate and voltage-gated Na+ channels • Ziconotide and voltage-gated N-type calcium channels • Naloxone and opioid receptors (partial antagonist) • Lidocaine and sodium channels • Gabapentin and VG-Ca2+ channels

which of the starling forces will be affected by the ff causes of edema?? Burns inflammation (due to release of histamine, cytokines) Arteriolar dilatation venous constriction ↑ venous pressure heart failure ECF volume expansion standing dec plasma protein concentration severe liver disease protein malnutrition nephrotic syndrome

↑ Filtration Coefficient Burns inflammation (due to release of histamine, cytokines) ↑ Capillary Hydrostatic Pressure Arteriolar dilatation venous constriction ↑ venous pressure heart failure ECF volume expansion standing ↓ Capillary Oncotic Pressure dec plasma protein concentration severe liver disease protein malnutrition nephrotic syndrome

What happens to the ff during exercise Venous PCO2 Pulmonary Blood Flow Arterial pH Arterial PO2 Arterial PCO2

↑ Venous PCO2 ↑ Pulmonary Blood Flow Arterial pH - ↑ in strenuous exercise (lactic acidosis); no change in moderate exercise Arterial PO2 - no change Arterial PCO2 - no change

Identify the kind of hypoxia present in the ff (Hypoxic Hypoxia, Anemic Hypoxia, Stagnant hypoxia, Histotoxic Hypoxia) ↓ cardiac output (+) Alveolar hypoventilation (high PaCO2) and hypoxemia (low PaO2) ↓ Hb (anemia) or ↓ saturation of hemoglobin with oxygen (SaO2) expected for a given PaO2 (e.g., CO poisoning or methemoglobinemia) impaired O2 extraction → ↓ CaO2-CvO2 and ↑ SVO2

↓ cardiac output - Stagnant hypoxia (+) Alveolar hypoventilation (high PaCO2) and hypoxemia (low PaO2) - Hypoxic Hypoxia ↓ Hb (anemia) or ↓ saturation of hemoglobin with oxygen (SaO2) expected for a given PaO2 (e.g., CO poisoning or methemoglobinemia) - Anemic Hypoxia impaired O2 extraction → ↓ CaO2-CvO2 and ↑ SVO2 - Histotoxic Hypoxia

When LDL enters cells by receptor-mediated endocytosis, which of the following does not occur? A. ↓ in the formation of cholesterol from mevalonic acid B. ↑in the intracellular concentration of cholesteryl esters C. ↑ in the transfer of cholesterol from the cell to HDL D. ↓ in the rate of synthesis of LDL receptors E. ↓ in the cholesterol in endosomes

↓ in the cholesterol in endosomes When LDL enters cells by receptor-mediated endocytosis, there will be: 1. Decrease in the formation of cholesterol and in the rate of synthesis of LDL receptors. 2. Increase in the intracellular concentration of cholesteryl esters, transfer of cholesterol from the cell to HDL, and amount of cholesterol in endosomes

lesion to what part of basal ganglia will lead to: ✓ inability to maintain posture? ✓ wild, flailing movements (hemiballismus)? ✓ quick, continuous, uncontrollable movements (e.g. in Huntington disease)? ✓ Tremors, cogwheel Rigidity, reduced voluntary movement (Akinesia), Postural problems?

✓ Globus Pallidus: inability to maintain posture ✓ Subthalamic Nucleus: wild, flailing movements (hemiballismus) ✓ Striatum: quick, continuous, uncontrollable movements (e.g. in Huntington disease) ✓ Substantia nigra: Tremors, cogwheel Rigidity, reduced voluntary movement (Akinesia), Postural problems

Chemicals and NTs involved in Pain Modulation ✓ Nucleus Raphe Magnum and Spinal Dorsal Horn? ✓ Locus Ceruleus? ✓ Periaqueductal gray matter? ✓ Spinal Dorsal Horn? ✓ Dorsal Root Ganglion?

✓ Nucleus Raphe Magnum and Spinal Dorsal Horn: Serotonin ✓ Locus Ceruleus: NE ✓ Periaqueductal gray matter: Morphine ✓ Spinal Dorsal Horn: Enkephalin ✓ Dorsal Root Ganglion: Opioids

4 causes of bronchoconstriction

✓ Parasympathetic Nervous System (cholinergic) ✓ Cool air, exercise ✓ Irritants (e.g., sulfur dioxide) ✓ Leukotrienes, Histamine

3 causes of bronchodilation

✓ Sympathetic Nervous system (adrenergic) ✓ Atropine ✓ Vasointestinal Peptide (VIP)


Conjuntos de estudio relacionados

PPT 18: Contracts - Defenses to Contract Enforceability - Voluntary Consent

View Set

Strategic Supply Chain MNGT CH. 13 MC

View Set

Chapter 1 the environment and sustainability

View Set

Biology Unit 1 Quiz: Chemistry of Life

View Set

Chapter 11-12 Personal Financial Planning

View Set